118
Question 1-3 of 147 Theme: Chest pain A. Pulmonary embolism B. Acute exacerbation asthma C. Physiological D. Mitral valve stenosis E. Aortic dissection F. Mitral regurgitation G. Bronchopneumonia H. Tuberculosis I. None of the above What is the most likely diagnosis for the scenario given? Each option may be used once, more than once or not at all. 1. A 28 year old Indian woman, who is 18 weeks pregnant, presents with increasing shortness of breath, chest pain and coughing clear sputum. She is apyrexial, blood pressure is 140/80 mmHg, heart rate 130 bpm and saturations 94% on 15L oxygen. On examination there is a mid diastolic murmur, there are bibasal crepitations and mild pedal oedema. She suddenly deteriorates and has a respiratory arrest. Her chest x-ray shows a whiteout of both of her lungs. You answered Pulmonary embolism The correct answer is Mitral valve stenosis Mitral stenosis is the commonest cause of cardiac abnormality occurring in pregnant women. Mitral stenosis is becoming less common in the UK population, however should be considered in women from countries where there is a higher incidence of rheumatic heart disease. Mitral stenosis causes a mid diastolic murmur which may be difficult to auscultate unless the patient is placed into the left lateral position. These patients are at risk of atrial fibrillation (up to 40%), which can also contribute to rapid decompensation such as pulmonary oedema (hence cxr 'whiteout' of lungs). Physiological changes in pregnancy may cause an otherwise asymptomatic patient to suddenly deteriorate. Balloon valvuloplasty is the treatment of choice. 2. A 28 year old woman, who is 30 weeks pregnant, presents with sudden onset chest pain associated with loss of consciousness. Her blood pressure is 170/90 mmHg, saturations on 15L oxygen 93%, heart rate 120 bpm and she is apyrexial. On examination there is an early diastolic murmur, occasional bibasal creptitations and mild peal oedema. An ECG shows ST elevation in leads II, III and aVF.

E. F. G. H. I. - 1 File Download

  • Upload
    others

  • View
    1

  • Download
    0

Embed Size (px)

Citation preview

Page 1: E. F. G. H. I. - 1 File Download

Question 1-3 of 147

Theme: Chest pain

A. Pulmonary embolism

B. Acute exacerbation asthma

C. Physiological

D. Mitral valve stenosis

E. Aortic dissection

F. Mitral regurgitation

G. Bronchopneumonia

H. Tuberculosis

I. None of the above What is the most likely diagnosis for the scenario given? Each option may be used once, more than once or not at all.

1. A 28 year old Indian woman, who is 18 weeks pregnant, presents with increasing shortness

of breath, chest pain and coughing clear sputum. She is apyrexial, blood pressure is 140/80

mmHg, heart rate 130 bpm and saturations 94% on 15L oxygen. On examination there is a

mid diastolic murmur, there are bibasal crepitations and mild pedal oedema. She suddenly

deteriorates and has a respiratory arrest. Her chest x-ray shows a whiteout of both of her

lungs.

You answered Pulmonary embolism

The correct answer is Mitral valve stenosis

Mitral stenosis is the commonest cause of cardiac abnormality occurring in pregnant

women. Mitral stenosis is becoming less common in the UK population, however should be

considered in women from countries where there is a higher incidence of rheumatic heart

disease. Mitral stenosis causes a mid diastolic murmur which may be difficult to auscultate

unless the patient is placed into the left lateral position. These patients are at risk of atrial

fibrillation (up to 40%), which can also contribute to rapid decompensation such as

pulmonary oedema (hence cxr 'whiteout' of lungs). Physiological changes in pregnancy may

cause an otherwise asymptomatic patient to suddenly deteriorate. Balloon valvuloplasty is

the treatment of choice.

2. A 28 year old woman, who is 30 weeks pregnant, presents with sudden onset chest pain

associated with loss of consciousness. Her blood pressure is 170/90 mmHg, saturations on

15L oxygen 93%, heart rate 120 bpm and she is apyrexial. On examination there is an early

diastolic murmur, occasional bibasal creptitations and mild peal oedema. An ECG shows

ST elevation in leads II, III and aVF.

Page 2: E. F. G. H. I. - 1 File Download

You answered Pulmonary embolism

The correct answer is Aortic dissection

Aortic dissection is associated with the 3rd trimester of pregnancy, connective tissue

disorders (Marfan's, Ehlers- Danlos) and bicuspid valve. Patients may complain of a tearing

chest pain or syncope. Clinically they may be hypertensive. The right coronary artery may

become involved in the dissection, causing myocardial infarct in up to 2% cases (hence ST

elevation in the inferior leads). An aortic regurgitant murmur may be auscultated.

3. A 28 year old woman, who is 18 weeks pregnant, presents with sudden chest pain. Her

blood pressure is 150/70 mmHg, saturations are 92% on 15L oxygen and her heart rate is

130 bpm. There are no murmurs and her chest is clear. There is signs of thrombophlebit is in

the left leg.

Pulmonary embolism

Chest pain, hypoxia and clear chest on auscultation in pregnancy should lead to a high

suspicion of pulmonary embolism.

Theme question in September 2011 exam Pregnant women can decompensate rapidly from cardiac compromise.

Please rate this question:

Discuss and give feedback

Next question

Chest pain in pregnancy

Aortic dissection

Predisposing factors in pregnancy are hypertension, congenital heart disease and Marfan's syndrome

Mainly Stanford type A dissections Sudden tearing chest pain, transient syncope Patient may be cold and clammy, hypertensive and have an aortic regurgitation murmur Involvement of the right coronary artery may cause inferior myocardial infarction

Surgical management

Page 3: E. F. G. H. I. - 1 File Download

Gestational timeframe Management

< 28/40 Aortic repair with the fetus kept in utero

28-32/40 Dependent on fetal condition

> 32/40 Primary Cesarean section followed by aortic repair at the same operation

Mitral stenosis

Most cases associated with rheumatic heart disease Becoming less common in British women; suspect in Immigrant women Commonest cardiac condition in pregnancy Commonly associated with mortality

Valve surgery; balloon valvuloplasty preferable

Pulmonary embolism

Leading cause of mortality in pregnancy Half dose scintigraphy; CT chest if underlying lung disease, should aid diagnosis

Treatment with low molecular weight heparin throughout pregnancy and 4-6 weeks after childbirth

Warfarin is contra indicated in pregnancy (though may be continued in women with mechanical heart valves due to the significant risk of thromboembolism)

References 1. Bates S.M. and Ginsberg J.S. How we manage venous thromboembolism during pregnancy. Blood2002 (100): 3470-3478. 2. Scarsbrook A.Fand Gleeson V. Investigating suspected pulmonary embolism in pregnancy. BMJ2007 (326) : 1135 doi: 10.1136/bmj.7399.1135. 3. Morley C. A. and Lim B. A. Lesson of the Week: The risks of delay in diagnosis of breathlessness in pregnancy. BMJ 1995 (311) : 1083.

Next question

Page 4: E. F. G. H. I. - 1 File Download

Question 4 of 147

A 67 year old man is investigated for biliary colic and a 4.8 cm abdominal aortic aneurysm is identified. Which of the following statements relating to this condition is untrue?

The wall will be composed of dense fibrous tissue only

The majority are located inferior to the renal arteries

They occur most often in current or former smokers

He should initially be managed by a process of active surveillance

Aortoduodenal fistula is a recognised complication following repair.

They are true aneurysms and have all 3 layers of arterial wall. Please rate this question:

Discuss and give feedback

Next question

Abdominal aorta aneurysm

Abdominal aortic aneurysms are a common problem in vascular surgery. They may occur as either true or false aneurysm. With the former all 3 layers of the arterial

wall are involved, in the latter only a single layer of fibrous tissue forms the aneurysm wall.

True abdominal aortic aneurysms have an approximate incidence of 0.06 per 1000 people. They are commonest in elderly men and for this reason the UK is now introducing the aneurysm screening program with the aim of performing an abdominal aortic ultrasound measurement in all men aged 65 years.

Causes

Several different groups of patients suffer from aneurysmal disease.

The commonest group is those who suffer from standard arterial disease, i.e. Those who arehypertensive, have diabetes and have been or are smokers.

Page 5: E. F. G. H. I. - 1 File Download

Other patients such as those suffering from connective tissue diseases such as Marfan's may also develop aneurysms. In patients with abdominal aortic aneurysms the extracellular matrix becomes disrupted with a change in the balance of collagen and elastic fibres.

Management

Most abdominal aortic aneurysms are an incidental finding.

Symptoms most often relate to rupture or impending rupture. 20% rupture anteriorly into the peritoneal cavity. Very poor prognosis. 80% rupture posteriorly into the retroperitoneal space The risk of rupture is related to aneurysm size, only 2% of aneurysms measuring less than

4cm in diameter will rupture over a 5 year period. This contrasts with 75% of aneurysms measuring over 7cm in diameter.

This is well explained by Laplaces' law which relates size to transmural pressure. For this reason most vascular surgeons will subject patients with an aneurysm size of 5cm or

greater to CT scanning of the chest, abdomen and pelvis with the aim of delineating anatomy and planning treatment. Depending upon co-morbidities, surgery is generally offered once the aneurysm is between 5.5cm and 6cm.

A CT reconstruction showing an infrarenal abdominal aortic aneurysm. The walls of the sac are

calcified which may facilitate identification on plain x-rays

Page 6: E. F. G. H. I. - 1 File Download

Image sourced from Wikipedia

Indications for surgery

Symptomatic aneurysms (80% annual mortality if untreated)

Increasing size above 5.5cm if asymptomatic Rupture (100% mortality without surgery)

Page 7: E. F. G. H. I. - 1 File Download

Surgical procedures Abdominal aortic aneurysm repair Procedure: GA Invasive monitoring (A-line, CVP, catheter) Incision: Midline or transverse Bowel and distal duodenum mobilised to access aorta. Aneurysm neck and base dissected out and prepared for cross clamp Systemic heparinisation Cross clamp (proximal first) Longitudinal aortotomy Atherectomy Deal with back bleeding from lumbar vessels and inferior mesenteric artery Insert graft either tube or bifurcated depending upon anatomy Suture using Prolene (3/0 for proximal , distal anastomosis suture varies according to site) Clamps off: End tidal CO2 will rise owing to effects of reperfusion, at this point major risk of myocardial events. Haemostasis Closure of aneurysm sac to minimise risk of aorto-enteric fistula Closure: Loop 1 PDS or Prolene to abdominal wall Skin- surgeons preference Post operatively: ITU (Almost all) Greatest risk of complications following emergency repair Complications: Embolic- gut and foot infarcts Cardiac - owing to premorbid states, re-perfusion injury and effects of cross clamp Wound problems Later risks related to graft- infection and aorto-enteric fistula Special groups

Supra renal AAA These patients will require a supra renal clamp and this carries a far higher risk of complications and risk of renal failure. Ruptured AAA

Pre-operatively the management depends upon haemodynamic instability. In patients with symptoms of rupture (typical pain, haemodynamic compromise and risk factors) then ideally prompt laparotomy. In those with vague symptoms and haemodynamic stability the ideal test is CT scan to determine whether rupture has occurred or not. Most common rupture site is retroperitoneal 80%. These patients will tend to develop retroperitoneal haematoma. This can be disrupted if Bp is allowed to rise too high so aim for Bp 100mmHg. Operative details are similar to elective repair although surgery should be swift, blind rushing often makes the situation worse. Plunging vascular clamps blindly into a pool of blood at the aneurysm neck carries the risk of injury the vena cava that these patients do not withstand. Occasionally a supracoeliac clamp is needed to effect temporary control, although leaving this applied for more than 20 minutes tends to carry a dismal outcome.

Page 8: E. F. G. H. I. - 1 File Download

EVAR Increasingly patients are now being offered endovascular aortic aneurysm repair. This is undertaken by surgeons and radiologists working jointly. The morphology of the aneurysm is important and not all are suitable. Here is a typical list of those features favoring a suitable aneurysm:

Long neck Straight iliac vessels Healthy groin vessels

Clearly few AAA patients possess the above and compromise has to be made. The use of fenestrated grafts can allow supra renal AAA to be treated. Procedure: GA Radiology or theatre Bilateral groin incisions Common femoral artery dissected out Heparinisation Arteriotomy and insertion of guide wire Dilation of arteriotomy Insertion of EVAR Device Once in satisfactory position it is released Arteriotomy closed once check angiogram shows good position and no endoleak Complications: Endoleaks depending upon site are either Type I or 2. These may necessitate re-intervention and all EVAR patients require follow up . Details are not needed for MRCS. References A reasonable review is provided by: Sakalihasan N, Limet R, Defawe O. Abdominal aortic aneurysm. Lancet 2005 (365):1577- 1589

Next question

Page 9: E. F. G. H. I. - 1 File Download

Question 5 of 147

Which of the following statements in relation to the p53 tumour suppressor protein is false?

It may induce necrosis of cells with non repairable DNA damage

It is affected in Li Fraumeni syndrome

It can induce DNA repair

It can halt the cell cycle

It may inhibit angiogenesis

When DNA cannot be repaired it will induce cellular apoptosis (not necrosis) Please rate this question:

Discuss and give feedback

Next question

Genetics and surgical disease

Some of the more commonly occurring genetic conditions occurring in surgical patients are presented here. Li-Fraumeni Syndrome

Autosomal dominant

Consists of germline mutations to p53 tumour suppressor gene High incidence of malignancies particularly sarcomas and leukaemias Diagnosed when:

*Individual develops sarcoma under 45 years *First degree relative diagnosed with any cancer below age 45 years and another family member develops malignancy under 45 years or sarcoma at any age BRCA 1 and 2

Page 10: E. F. G. H. I. - 1 File Download

Carried on chromosome 17 (BRCA 1) and Chromosome 13 (BRCA 2) Linked to developing breast cancer (60%) risk. Associated risk of developing ovarian cancer (55% with BRCA 1 and 25% with BRCA 2).

Lynch Syndrome

Autosomal dominant Develop colonic cancer and endometrial cancer at young age 80% of affected individuals will get colonic and/ or endometrial cancer

High risk individuals may be identified using the Amsterdam criteria

Amsterdam criteria

Three or more family members with a confirmed diagnosis of colorectal cancer, one of whom is a first degree (parent, child, sibling) relative of the other two. Two successive affected generations. One or more colon cancers diagnosed under age 50 years. Familial adenomatous polyposis (FAP) has been excluded. Gardners syndrome

Autosomal dominant familial colorectal polyposis

Multiple colonic polyps Extra colonic diseases include: skull osteoma, thyroid cancer and epidermoid cysts Desmoid tumours are seen in 15% Mutation of APC gene located on chromosome 5 Due to colonic polyps most patients will undergo colectomy to reduce risk of colorectal

cancer

Now considered a variant of familial adenomatous polyposis coli

Next question

Page 11: E. F. G. H. I. - 1 File Download

Question 6 of 147

Which of the following cell types is most likely to be identified in the wall of a fistula in ano?

Squamous cells

Goblet cells

Columnar cells

Ciliated columnar cells

None of the above

A fistula is an abnormal connection between two epithelial lined surfaces, in the case of a fistula in ano it will be lined by squamous cells. Please rate this question:

Discuss and give feedback

Next question

Fistulas

A fistula is defined as an abnormal connection between two epithelial surfaces. There are many types ranging from Branchial fistulae in the neck to entero-cutaneous

fistulae abdominally.

In general surgical practice the abdominal cavity generates the majority and most of these arise from diverticular disease and Crohn's.

As a general rule all fistulae will resolve spontaneously as long as there is no distal obstruction. This is particularly true of intestinal fistulae.

The four types of fistulae are: Enterocutaneous These link the intestine to the skin. They may be high (>500ml) or low output (<250ml) depending upon source. Duodenal /jejunal fistulae will tend to produce high volume, electrolyte rich secretions which can lead to severe excoriation of the skin. Colo-cutaneous fistulae will tend to leak faeculent material. Both fistulae may result from the spontaneous rupture of an abscess cavity onto the skin (such as following perianal abscess drainage) or may occur as a result of iatrogenic input. In some

Page 12: E. F. G. H. I. - 1 File Download

cases it may even be surgically desirable e.g. mucous fistula following sub total colectomy for colitis. Suspect if there is excess fluid in the drain. Enteroenteric or Enterocolic

This is a fistula that involves the large or small intestine. They may originate in a similar manner to enterocutaneous fistulae. A particular problem with this fistula type is that bacterial overgrowth may precipitate malabsorption syndromes. This may be particularly serious in inflammatory bowel disease. Enterovaginal Aetiology as above. Enterovesicular This type of fistula goes to the bladder. These fistulas may result in frequent urinary tract infections, or the passage of gas from the urethra during urination. Management

Some rules relating to fistula management:

They will heal provided there is no underlying inflammatory bowel disease and no distal obstruction, so conservative measures may be the best option

Where there is skin involvement, protect the overlying skin, often using a well fitted stoma bag- skin damage is difficult to treat

A high output fistula may be rendered more easily managed by the use of octreotide, this will tend to reduce the volume of pancreatic secretions.

Nutritional complications are common especially with high fistula (e.g. high jejunal or duodenal) these may necessitate the use of TPN to provide nutritional support together with the concomitant use of octreotide to reduce volume and protect skin.

When managing perianal fistulae surgeons should avoid probing the fistula where acute inflammation is present, this almost always worsens outcomes.

When perianal fistulae occur secondary to Crohn's disease the best management option is often to drain acute sepsis and maintain that drainage through the judicious use of setons whilst medical management is implemented.

Always attempt to delineate the fistula anatomy, for abscesses and fistulae that have an intra abdominal source the use of barium and CT studies should show a track. For perianal fistulae surgeons should recall Goodsall's rule in relation to internal and external openings.

Next question

Page 13: E. F. G. H. I. - 1 File Download

Question 7 of 147

A 22 year old man is referred to the surgical clinic. He has been complaining of varicose veins for many years. On examination he has extensive varicosities of the right leg, there are areas of marked port wine staining. The saphenofemoral junction is competent on doppler assessment. The most likely underlying diagnosis is:

Deep vein thrombosis

Klippel-Trenaunay syndrome

Varicose veins due to sapheno-popliteal junction incompetence

Sturge - Weber syndrome

Angiosarcoma

Sturge - Weber syndrome is a an arteriovenous malformation affecting the face and CNS, the peripheral vessels are not affected. Simple varicose veins should not typically be associated with port wine staining, nor should a DVT or angiosarcoma. Please rate this question:

Discuss and give feedback

Next question

Klippel-Trenaunay syndrome

Klippel-Trenaunay-Weber syndrome generally affects a single extremity, although cases of multiple affected limbs have been reported. The leg is the most common site followed by the arms, the trunk, and rarely the head and the neck Signs and symptoms The birth defect is diagnosed by the presence of a combination of these symptoms:

One or more distinctive port-wine stains with sharp borders

Varicose veins Hypertrophy of bony and soft tissues, that may lead to local gigantism or shrinking. An improperly developed lymphatic system

Page 14: E. F. G. H. I. - 1 File Download

In some cases, port-wine stains (capillary port wine type) may be absent. Such cases are very rare and may be classified as "atypical Klippel-Trenaunay syndrome". KTS can either affect blood vessels, lymph vessels, or both. The condition most commonly presents with a mixture of the two. Those with venous involvement experience increased pain and complications.

Next question

Page 15: E. F. G. H. I. - 1 File Download

Question 8 of 147

A 68 year old man presents with an ulcerated lesion on his right cheek. It is excised and on histological assessment a squamous cell carcinoma is diagnosed. It measures 25mm in diameter and is 4mm deep. Which of the following statements relating to this condition is false?

In this particular case margins of at least 6mm are required

Use of cryosurgery to treat this patients lesion would have been unsafe

Use of radiotherapy to treat this lesion would have been unsafe

This patients local recurrence rate may approach 15%

The disease usually spreads via lymphatics

Poor prognostic factors in SCC:

Size >20mm (local recurrence rate of up to 15%) Depth greater than 4mm (risk of metastasis up to 30%)

This man has an SCC with significant risk of metastasis. Although cryotherapy may be used to treat SCC it would be most unsafe in this setting as the lesion extends deeply. However, radiotherapy is a safe treatment modality for SCC and may be used in selected cases. It is unwise to use radiotherapy in areas prone to radionecrosis e.g. the nose. Please rate this question:

Discuss and give feedback

Next question

Squamous cell carcinoma of the skin

Second most common skin malignancy Derived from epidermal keratinocytes Commonest in fair skinned individuals in sun exposed sites May occur in perianal and genital skin especially in association with Human Papilloma Virus

16 and 18 infections.

Page 16: E. F. G. H. I. - 1 File Download

Groups at high risk

Renal transplant and on immunosuppression

Individuals with HIV

Those who have received psoralen UVA therapy

Chronic wounds (Marjolins ulcer)

Xeroderma pigmentosum

Oculocutaneous albinism

Prognosis

Good Prognosis Poor prognosis

Well differentiated tumours Poorly differentiated tumours

<20mm diameter >20mm in diameter

<2mm deep >4mm deep

No associated diseases Immunosupression for whatever reason

Treatment Surgical excision with 4mm margins if lesion <20mm in diameter. If tumour >20mm then margins should be 6mm. Squamous cell carcinomas of the skin typically arise in areas of sun exposure as shown here

Page 17: E. F. G. H. I. - 1 File Download

Image sourced from Wikipedia

References The British Association of Dermatology provides guidelines for the diagnosis and treatment of SCC. http://www. bad.org.uk

Next question

Page 18: E. F. G. H. I. - 1 File Download

Question 9 of 147

A 23 year old man presents with weight loss fatigue and lymphadenopathy. He is diagnosed with

tuberculosis. Which of the following processes most closely matches the underlying pathological

process?

Type 1 hypersensitivity reaction

Type 2 hypersensitivity reaction

Type 3 hypersensitivity reaction

Type 4 hypersensitivity reaction

None of the above

Granulomas (which occur in tuberculosis) are a feature of Type 4 hypersensitivity reactions.

Please rate this question:

Discuss and give feedback

Next question

Hypersensitivity reactions

The Gell and Coombs classification divides hypersensitivity reactions into 4 types

Type I Type II Type III Type IV

Page 19: E. F. G. H. I. - 1 File Download

Type I Type II Type III Type IV

Description Anaphylactic Cytotoxic Immune

complex

Delayed type

Mediator IgE IgG, IgM IgG, Ig A, IgM T-cells

Antigen Exogenous Cell surface Soluble Tissues

Response

time

Minutes Hours Hours 2-3 days

Examples Asthma

Hay fever

Autoimmune haemolytic

anaemia

Pemphigus

Goodpasture's

Serum sickness

SLE

Aspergillosis

Graft versus host

disease

Contact dermatitis

Next question

Page 20: E. F. G. H. I. - 1 File Download

Question 10 of 147

A 73 year old man undergoes an emergency amputation for severe lower limb sepsis and gangrene. Post operatively he develops disseminated intravascular coagulation. Which of the following clotting factors will be most rapidly consumed in this process?

Factor V and VIII

Factor I

Factor I and III

Factor III and VII

Factor VI and VIII

D-I-S-S-E-M-I-N-A-T-E-D D-Dx: D dimer I-Immune complexes S-Snakebite, shock, heatstroke S-SLE E-Eclampsia, HELLP syndrome M-Massive tissue damage I-Infections: viral and bacterial N-Neoplasms A-Acute promyelocytic leukemia T-Tumor products: Tissue Factor (TF) and TF-like factors released by carcinomas of pancreas, prostate, lung, colon, stomach E-Endotoxins (bacterial) D-Dead fetus (retained)

DIC Will tend to consume factors five and eight intially (and platelets). Please rate this question:

Discuss and give feedback

Next question

Disseminated intravascular coagulation

Page 21: E. F. G. H. I. - 1 File Download

Simultaneous coagulation and haemorrhage caused by initially formation of thrombi which consume clotting factors (factors 5,8) and platelets, ultimately leading to bleeding Causes include:

Infection Malignancy Trauma e.g. major surgery, burns, shock, dissecting aortic aneurysm Liver disease

Obstetric complications

Key points

Clinically bleeding is usually a dominant feature, bruising, ischaemia and organ failure Blood tests: prolonged clotting times, thrombocytopenia, decreased fibrinogen, increased

fibrinogen degradation products Treat the underlying cause and supportive management

Next question

Page 22: E. F. G. H. I. - 1 File Download

Question 11 of 147

A pregnant women suddenly develops bilateral leg swelling. Her mother and aunt were troubled by the same problem. What is the most likely underlying abnormality?

Anti endomysial antibodies

Anti nuclear antibodies

Anti cardiolipin antibodies

Anti thyroid antibodies

Anti mitochondrial antibodies

Anti phospholipid syndrome= following antibodies

Lupus anticoagulant

Anti-cardiolipin Anti-β2-glycoprotein

Theme from September 2014 Exam Antiphospholipid syndrome, is an autoimmune, hypercoagulable state caused by antiphospholipid antibodies. APS provokes blood clots (thrombosis) in both arteries and veins as well as pregnancy-related complications such as miscarriage, stillbirth, preterm delivery, or severe preeclampsia. The diagnostic criteria requires one clinical event, i.e. thrombosis or pregnancy complication, and two positive blood tests spaced at least 3 months apart. These antibodies are: lupus anticoagulant, anti-cardiolipin and anti-β2-glycoprotein. Antiphospholipid syndrome can be primary or secondary. Primary antiphospholipid syndrome occurs in the absence of any other related disease. Secondary antiphospholipid syndrome occurs with other autoimmune diseases, such as systemic lupus erythematosus (SLE). In rare cases, APS leads to rapid organ failure due to generalised thrombosis; this is termed "catastrophic antiphospholipid syndrome" (CAPS) and is associated with a high risk of death. Antiphospholipid syndrome often requires treatment with anticoagulant medication such as heparin to reduce the risk of further episodes of thrombosis and improve the prognosis of pregnancy. Warfarin is not used during pregnancy because it can cross the placenta, unlike heparin, and is teratogenic. Please rate this question:

Page 23: E. F. G. H. I. - 1 File Download

Discuss and give feedback

Next question

Hypercoagulability

Type of thrombophilia Features

Antithrombin deficiency Antithrombin inactivates thrombin and factor XII a, XIa, IXa and Xa

Rare defect, inherited in autosomal dominant fashion

10x increase in risk of thrombotic events Heparin may be ineffective because it works via antithrombin

Protein C and S

deficiency

These are natural anticoagulants (vitamin K dependent synthesis)

Protein C produced by liver

Protein S produced by liver, megakaryocytes, Leydig cells and

endothelial cells

Protein C and S bind to form activated complex which binds to factor

V Deficiency accounts for up to 5% of thrombotic episodes

Factor V Leiden Resistance to anticoagulant effect of activated protein C

May account for up to 20% or more of thrombotic episodes

Prevalence of 7% in Europe

Most common genetic defect accounting for DVT

Antiphospholipid syndrome

Multi organ disease

Pregnancy involvement common

Arterial and venous thromboses

Either Lupus anticoagulant or Anti cardiolipin antibodies

APTT usually prolonged

Antibodies may be elevated following surgery, drugs or malignancy

Need anticoagulation with INR between 3 and 4

Next question

Page 24: E. F. G. H. I. - 1 File Download

Question 12 of 147

A 63 year old Japanese man presents with epigastric discomfort and iron deficiency anaemia. He undergoes an upper GI endoscopy, where the following appearances are found:

Image sourced from Wikipedia

The most likely diagnosis is:

Squamous cell carcinoma

Linitis plastica

Leiomyosarcoma

Gastric varices

None of the above

During upper GI endoscopy, a linitis plastica lesion may prevent gastric distension.

Page 25: E. F. G. H. I. - 1 File Download

Linitis plastica produces a diffuse infiltrating lesion, the stomach is fibrotic and rigid and will not typically distend. This may be described as a 'leather bottle stomach'. Diagnosis is made with a combination of pathology examination with endoscopy, radiological or surgical assessment. Pathologically signet-ring cell proliferation occurs. Please rate this question:

Discuss and give feedback

Next question

Gastric cancer

Overview

There are 700,000 new cases of gastric cancer worldwide each year. It is most common in Japan and less common in western countries. It is more common in men and incidence rises with increasing age. The exact cause of many sporadic cancer is not known, however, familial cases do occur in HNPCC families. In addition, smoking and smoked or preserved foods increase the risk. Japanese migrants retain their increased risk (decreased in subsequent generations). The distribution of the disease in western countries is changing towards a more proximal location (perhaps due to rising obesity). Pathology There is some evidence of support a stepwise progression of the disease through intestinal metaplasia progressing to atrophic gastritis and subsequent dysplasia, through to cancer. The favoured staging system is TNM. The risk of lymph node involvement is related to size and depth of invasion; early cancers confined to submucosa have a 20% incidence of lymph node metastasis. Tumours of the gastro-oesophageal junction are classified as below:

Type

1

True oesophageal cancers and may be associated with Barrett's oesophagus.

Type

2

Carcinoma of the cardia, arising from cardiac type epithelium or short segments with intestinal metaplasia at the oesophagogastric junction.

Type

3

Sub cardial cancers that spread across the junction. Involve similar nodal stations to gastric cancer.

Groups for close endoscopic monitoring

Intestinal metaplasia of columnar type

Atrophic gastritis Low to medium grade dysplasia Patients who have previously undergone resections for benign peptic ulcer disease (except

highly selective vagotomy).

Page 26: E. F. G. H. I. - 1 File Download

Referral to endoscopy

Patients of any age with

dyspepsia and any of the

following

Patients without

dyspepsia

Worsening dyspepsia

Chronic gastrointestinal bleeding Dysphagia Barretts oesophagus

Dysphagia Unexplained abdominal

pain or weight loss

Intestinal metaplasia

Weight loss Vomiting Dysplasia

Iron deficiency anaemia Upper abdominal mass Atrophic gastritis

Upper abdominal mass Jaundice Patient aged over 55 years with

unexplained or persistent dyspepsia

Upper GI endoscopy performed for dyspepsia. The addition of dye spraying (as shown in the bottom

right) may facilitate identification of smaller tumours

Page 27: E. F. G. H. I. - 1 File Download

Image sourced from Wikipedia

Staging

CT scanning of the chest abdomen and pelvis is the routine first line staging investigation in most centres.

Laparoscopy to identify occult peritoneal disease PET CT (particularly for junctional tumours)

Treatment

Proximally sited disease greater than 5-10cm from the OG junction may be treated by sub total gastrectomy

Total gastrectomy if tumour is <5cm from OG junction For type 2 junctional tumours (extending into oesophagus) oesophagogastrectomy is usual Endoscopic sub mucosal resection may play a role in early gastric cancer confined to the

mucosa and perhaps the sub mucosa (this is debated) Lymphadenectomy should be performed. A D2 lymphadenectomy is widely advocated by the

Japanese, the survival advantages of extended lymphadenectomy have been debated. However, the overall recommendation is that a D2 nodal dissection be undertaken.

Most patients will receive chemotherapy either pre or post operatively.

Prognosis

Page 28: E. F. G. H. I. - 1 File Download

UK Data

Disease extent Percentage 5 year survival

All RO resections 54%

Early gastric cancer 91%

Stage 1 87%

Stage 2 65%

Stage 3 18%

Operative procedure Total Gastrectomy , lymphadenectomy and Roux en Y anastomosis General anaesthesia Prophylactic intravenous antibiotics Incision: Rooftop. Perform a thorough laparotomy to identify any occult disease. Mobilise the left lobe of the liver off the diaphragm and place a large pack over it. Insert a large self retaining retractor e.g. omnitract or Balfour (take time with this, the set up should be perfect). Pack the small bowel away. Begin by mobilising the omentum off the transverse colon. Proceed to detach the short gastric vessels. Mobilise the pylorus and divide it at least 2cm distally using a linear cutter stapling device. Continue the dissection into the lesser sac taking the lesser omentum and left gastric artery flush at its origin. The lymph nodes should be removed en bloc with the specimen where possible. Place 2 stay sutures either side of the distal oesophagus. Ask the anaesthetist to pull back on the nasogastric tube. Divide the distal oesophagus and remove the stomach. The oesphago jejunal anastomosis should be constructed. Identify the DJ flexure and bring a loop of jejunum up to the oesophagus (to check it will reach). Divide the jejunum at this point. Bring the divided jejunum either retrocolic or antecolic to the oesophagus. Anastamose the oesophagus to the jejunum, using either interrupted 3/0 vicryl or a stapling device. Then create the remainder of the Roux en Y reconstruction distally. Place a jejunostomy feeding tube. Wash out the abdomen and insert drains (usually the anastomosis and duodenal stump). Help the anaesthetist insert the nasogastric tube (carefully!) Close the abdomen and skin.

Page 29: E. F. G. H. I. - 1 File Download

Enteral feeding may commence on the first post-operative day. However, most surgeons will leave patients on free NG drainage for several days and keep them nil by mouth.

Next question

Page 30: E. F. G. H. I. - 1 File Download

0 / 3 Question 13-15 of 147

Theme: Genetics and cancer

A. Multiple endocrine neoplasia type I

B. Multiple endocrine neoplasia type II

C. Gardner's syndrome

D. Lynch Syndrome

E. Kartagener's syndrome

F. Neurofibromatosis Type I

G. Neurofibromatosis Type II Please select the most likely condition for the disease process described. Each option may be used once, more than once or not at all.

13. A 40 year old male is found to have multiple colonic polyps during a colonoscopy. He

mentions that he has extra teeth.

You answered Multiple endocrine neoplasia type I

The correct answer is Gardner's syndrome

Gardner's syndrome is an AD disorder, characterised by: Colonic polyps, supernumerary

teeth, jaw osteomas, congenital hypertrophy of retinal pigment. osteomas of the skull,

thyroid cancer, epidermoid cysts, fibromas and sebaceous cysts.

14. A 10 year old boy who has learning difficulties, reports a difference in size between his

two legs.

You answered Multiple endocrine neoplasia type I

The correct answer is Neurofibromatosis Type I

Neurofibromatosis type I. A hallmark finding is a plexiform neurofibroma, which is a

sheet of neurofibromatosis tissue which encases major nerves. In children this attracts

extra blood circulation, which can accelerate growth of the affected limb.

Other features include:

Schwannoma, > 6

Cafe au lait spots, axillary freckling, Lisch nodules, Optic glioma. Meningiomas, Glioma,

or Schwannoma.

15. A 22 year old is found to have bilateral acoustic neuromas.

Page 31: E. F. G. H. I. - 1 File Download

You answered Multiple endocrine neoplasia type I

The correct answer is Neurofibromatosis Type II

In NF2 bilateral acoustic neuromas are characteristic with a family history of

Neurofibroma,

Please rate this question:

Discuss and give feedback

Next question

Genetics and surgical disease

Some of the more commonly occurring genetic conditions occurring in surgical patients are presented here. Li-Fraumeni Syndrome

Autosomal dominant Consists of germline mutations to p53 tumour suppressor gene High incidence of malignancies particularly sarcomas and leukaemias Diagnosed when:

*Individual develops sarcoma under 45 years *First degree relative diagnosed with any cancer below age 45 years and another family member develops malignancy under 45 years or sarcoma at any age BRCA 1 and 2

Carried on chromosome 17 (BRCA 1) and Chromosome 13 (BRCA 2)

Linked to developing breast cancer (60%) risk. Associated risk of developing ovarian cancer (55% with BRCA 1 and 25% with BRCA 2).

Lynch Syndrome

Autosomal dominant

Develop colonic cancer and endometrial cancer at young age 80% of affected individuals will get colonic and/ or endometrial cancer High risk individuals may be identified using the Amsterdam criteria

Page 32: E. F. G. H. I. - 1 File Download

Amsterdam criteria

Three or more family members with a confirmed diagnosis of colorectal cancer, one of whom is a first degree (parent, child, sibling) relative of the other two. Two successive affected generations. One or more colon cancers diagnosed under age 50 years. Familial adenomatous polyposis (FAP) has been excluded. Gardners syndrome

Autosomal dominant familial colorectal polyposis

Multiple colonic polyps Extra colonic diseases include: skull osteoma, thyroid cancer and epidermoid cysts Desmoid tumours are seen in 15% Mutation of APC gene located on chromosome 5 Due to colonic polyps most patients will undergo colectomy to reduce risk of colorectal

cancer

Now considered a variant of familial adenomatous polyposis coli

Next question

Page 33: E. F. G. H. I. - 1 File Download

Question 16 of 147

A 52 year old lady is referred to the breast clinic with symptoms of nipple discharge. The discharge is usually thick and green. Which of the following statements relating to the most likely underlying diagnosis is untrue?

The majority of patients will be smokers

Typically produces blood stained nipple discharge

It is not associated with increased risk of breast cancer

May result in development of mammary duct fistula

May require total duct excision (Hadfields operation) if it fails to resolve

Blood stained nipple discharge should always be investigated. Nipple fluid cytology is generally unhelpful.

Discharge of this type of material is most likely to be due to duct ectasia. Green or brown discharge is most common. Blood stained discharge should raise concern of intraductal papilloma or cancer. Please rate this question:

Discuss and give feedback

Next question

Nipple discharge

Causes of nipple discharge

Physiological During breast feeding

Galactorrhoea Commonest cause may be response to emotional events, drugs such as histamine receptor antagonists are also implicated

Hyperprolactinaemia Commonest type of pituitary tumour

Microadenomas <1cm in diameter

Page 34: E. F. G. H. I. - 1 File Download

Macroadenomas >1cm in diameter

Pressure on optic chiasm may cause bitemporal hemianopia

Mammary duct

ectasia

Dilatation breast ducts.

Most common in menopausal women

Discharge typically thick and green in colour

Most common in smokers

Carcinoma Often blood stained

May be underlying mass or axillary lymphadenopathy

Intraductal papilloma Commoner in younger patients

May cause blood stained discharge

There is usually no palpable lump

Assessment of patients

Examine breast and determine whether there is mass lesion present

All mass lesions should undergo Triple assessment.

Reporting of investigations Where a mass lesion is suspected or investigations are requested these are prefixed using a system that denotes the investigation type e.g. M for mammography, followed by a numerical code as shown below:

1 No abnormality

2 Abnormality with benign features

3 Indeterminate probably benign

4 Indeterminate probably malignant

5 Malignant

Page 35: E. F. G. H. I. - 1 File Download

Management of non malignant nipple discharge

Exclude endocrine disease Nipple cytology unhelpful

Smoking cessation advice for duct ectasia For duct ectasia with severe symptoms, total duct excision may be warranted.

Next question

Page 36: E. F. G. H. I. - 1 File Download

Question 17 of 147

Which of the following statements relating to gas gangrene is untrue?

There is necrosis with putrefaction

The causative pathogens may be detected on normal perineal skin

Treatment with low dose penicillin is indicated

Hyperbaric oxygen may be beneficial

Clostridium perfringens is a recognised cause

Rapid surgery and high dose antibiotics are indicated in the treatment of gas gangrene. Please rate this question:

Discuss and give feedback

Next question

Meleney's Gangrene and Necrotising Fasciitis

Necrotising fasciitis

Advancing soft tissue infection associated with fascial necrosis

Uncommon, but can be fatal In many cases there is underlying background immunosuppression e.g. Diabetes Caused by polymicrobial flora (aerobic and anaerobic) and MRSA is seen increasingly in

cases of necrotising fasciitis

Streptococcus is the commonest organism in isolated pathogen infection (15%)

Meleneys gangrene

Meleneys is a similar principle but the infection is more superficially sited than necrotising fasciitis and often confined to the trunk

Page 37: E. F. G. H. I. - 1 File Download

Fournier gangrene

Necrotising fasciitis affecting the perineum Polymicrobial with E-coli and Bacteroides acting in synergy

Clinical features

Fever Pain Cellulitis Oedema Induration Numbness Late findings Purple/black skin discolouration Blistering Haemorrhagic bullae Crepitus Dirty Dishwater fluid discharge Septic shock A typical case of gas gangrene presenting late demonstrating some of the features described above

Image sourced from Wikipedia

Diagnosis is mainly clinical Management

Page 38: E. F. G. H. I. - 1 File Download

Radical surgical debridement forms the cornerstone of management Sterile dressing is used to dress the wound Reconstructive surgery is considered once the infection is completely treated

Reference Hasham S, Matteucci P, Stanley PR, Hart NB. Necrotising fasciitis. BMJ 2005;330:830-833.

Next question

Page 39: E. F. G. H. I. - 1 File Download

Question 18 of 147

A 30 year old man presents with abdominal distension, a laparotomy is performed, at operation the abdomen is filled with a large amount of gelatinous fluid. What is the most likely underlying diagnosis?

Infection with entamoeba histolytica

Pseudomxyoma peritonei

Metastatic colonic cancer

Chylous ascites

None of the above

Pseudomyxoma is associated with the deposition of large amounts of gelatinous material. The appendix is the commonest organ or origin. Please rate this question:

Discuss and give feedback

Next question

Pseudomyxoma Peritonei

Rare mucinous tumour Most commonly arising from the appendix (other abdominal viscera are also recognised as

primary sites) Incidence of 1-2/1,000,000 per year The disease is characterised by the accumulation of large amounts of mucinous material in

the abdominal cavity

Treatment Is usually surgical and consists of cytoreductive surgery (and often peritonectomy c.f Sugarbaker procedure) combined with intra peritoneal chemotherapy with mitomycin C. Survival is related to the quality of primary treatment and in Sugarbakers own centre 5 year survival rates of 75% have been quoted. Patients with disseminated intraperitoneal malignancy from another

Page 40: E. F. G. H. I. - 1 File Download

source fare far worse. In selected patients a second look laparotomy is advocated and some practice this routinely.

Next question

Page 41: E. F. G. H. I. - 1 File Download

Question 19 of 147

A 30 year old man is suspected of having appendicitis. At operation an inflamed Meckels diverticulum is found. Which of the following vessels is responsible for the blood supply to a Meckels diverticulum?

Right colic artery

Vitelline artery

Appendicular artery

Internal iliac artery

External iliac artery

The vitelline arteries supply a Meckels these are usually derived from the ileal arcades. Please rate this question:

Discuss and give feedback

Next question

Meckel's diverticulum

Congenital abnormality resulting in incomplete obliteration of the vitello-intestinal duct

Normally, in the foetus, there is an attachment between the vitello-intestinal duct and the yolk sac.This disappears at 6 weeks gestation.

The tip is free in majority of cases. Associated with enterocystomas, umbilical sinuses, and omphaloileal fistulas. Arterial supply: omphalomesenteric artery. 2% of population, 2 inches long, 2 feet from the ileocaecal valve.

Typically lined by ileal mucosa but ectopic gastric mucosa can occur, with the risk of peptic ulceration. Pancreatic and jejunal mucosa can also occur.

Clinical

Normally asymptomatic and an incidental finding.

Complications are the result of obstruction, ectopic tissue, or inflammation.

Page 42: E. F. G. H. I. - 1 File Download

Removal if narrow neck or symptomatic. Options are between wedge excision or formal small bowel resection and anastomosis.

Next question

Page 43: E. F. G. H. I. - 1 File Download

Question 20 of 147

Which of the following associations are incorrect?

Afro-Caribbean skin and keloid scarring

Extensive third degree burns and wound contraction

Chemotherapy and dehisence of healed wounds

Poor healing at the site of previous radiotherapy

Zinc deficiency and delayed healing

Please rate this question:

Discuss and give feedback

Next question

Wound healing

Surgical wounds are either incisional or excisional and either clean, clean contaminated or dirty. Although the stages of wound healing are broadly similar their contributions will vary according to the wound type. The main stages of wound healing include: Haemostasis

Minutes to hours following injury Vasospasm in adjacent vessels, platelet plug formation and generation of fibrin rich clot.

Inflammation

Typically days 1-5 Neutrophils migrate into wound (function impaired in diabetes).

Page 44: E. F. G. H. I. - 1 File Download

Growth factors released, including basic fibroblast growth factor and vascular endothelial growth factor.

Fibroblasts replicate within the adjacent matrix and migrate into wound.

Macrophages and fibroblasts couple matrix regeneration and clot substitution.

Regeneration

Typically days 7 to 56

Platelet derived growth factor and transformation growth factors stimulate fibroblasts and epithelial cells.

Fibroblasts produce a collagen network. Angiogenesis occurs and wound resembles granulation tissue.

Remodeling

From 6 weeks to 1 year Longest phase of the healing process and may last up to one year (or longer). During this phase fibroblasts become differentiated (myofibroblasts) and these facilitate

wound contraction.

Collagen fibres are remodeled. Microvessels regress leaving a pale scar.

The above description represents an idealised scenario. A number of diseases may distort this process. Neovascularisation is an important early process. Endothelial cells may proliferate in the wound bed and recanalise to form a vessel. Vascular disease, shock and sepsis can all compromise microvascular flow and impair healing. Conditions such as jaundice will impair fibroblast synthetic function and immunity with a detrimental effect in most parts of the healing process. Problems with scars: Hypertrophic scars Excessive amounts of collagen within a scar. Nodules may be present histologically containing randomly arranged fibrils within and parallel fibres on the surface. The tissue itself is confined to the extent of the wound itself and is usually the result of a full thickness dermal injury. They may go on to develop contractures. Image of hypertrophic scarring. Note that it remains confined to the boundaries of the original wound:

Page 45: E. F. G. H. I. - 1 File Download

Image sourced from Wikipedia

Keloid scars

Excessive amounts of collagen within a scar. Typically a keloid scar will pass beyond the boundaries of the original injury. They do not contain nodules and may occur following even trivial injury. They do not regress over time and may recur following removal. Image of a keloid scar. Note the extension beyond the boundaries of the original incision:

Image sourced from Wikipedia

Drugs which impair wound healing:

Page 46: E. F. G. H. I. - 1 File Download

Non steroidal anti inflammatory drugs Steroids Immunosupressive agents Anti neoplastic drugs

Closure

Delayed primary closure is the anatomically precise closure that is delayed for a few days but before granulation tissue becomes macroscopically evident. Secondary closure refers to either spontaneous closure or to surgical closure after granulation tissue has formed.

Next question

Page 47: E. F. G. H. I. - 1 File Download

Question 21 of 147

A 45 year old women is identified as having a gastric gastro-intestinal stromal tumour. What is the

usual cell of origin of these lesions?

Brunners glands

Interstitial cells of Cajal

Primitive stem cells of the gut wall

Fundic glands

Antral goblet cells

GIST's are derived from the interstitial pacemaker cells of Cajal. This means that they are often

located extramucosally and macroscopically, demonstrate little mucosal disruption.

Please rate this question:

Discuss and give feedback

Next question

Gastrointestinal stromal tumour

GIST's are not common tumours (10 per million) and originate primarily from the interstitial

pacemaker cells (of Cajal). Up to 70% occur in the stomach, the remainder occurring in the small

intestine (20%) and the colon and rectum (5%). Up to 95% are solitary lesions and most are

sporadic. The vast majority express CD117 which is a transmembrane tyrosine kinase receptor and

in these there is a mutation of the c-KIT gene.

Page 48: E. F. G. H. I. - 1 File Download

The goal of surgery is resection of the tumour with a 1-2cm margin of normal tissue. As a result

extensive resections are not required. Unfortunately there is a high local recurrence rate, the risk of

which is related to site, incomplete resections and high mitotic count. Salvage surgery for recurrent

disease is associated with a median survival of 15 months.

The prognosis in high risk patients is greatly improved through the use of imatinib, which in the

ACOSOG trial (imatinib vs placebo) improved relapse rates from 17% to 2%.

In the UK it is advocated by NICE for use in patients with metastatic disease or locally unresectable

disease.

Next question

Page 49: E. F. G. H. I. - 1 File Download

Question 22 of 147

A 23 year old man fractures his right tibia in a sporting accident. At which point in the healing process is fracture callus most likely to be visible radiologically?

1 day

7 days

8 weeks

6 weeks

3 weeks

Fracture callus is composed of fibroblasts and chondroblasts and the synthesis of fibrocartilage. It is typically visible on radiographs approximately 3 weeks following injury. If delayed then there may be risk of non union. Please rate this question:

Discuss and give feedback

Next question

Fracture healing

Bone fracture - Bleeding vessels in the bone and periosteum - Clot and haematoma formation - The clot organises over a week (improved structure and collagen) - The periosteum contains osteoblasts which produce new bone - Mesenchymal cells produce cartilage (fibrocartilage and hyaline cartilage) in the soft tissue around the fracture - Connective tissue + hyaline cartilage = callus - As the new bone approaches the new cartilage, endochondral ossification occurs to bridge the gap - Trabecular bone forms - Trabecular bone is resorbed by osteoclasts and replaced with compact bone Factors affecting fracture healing

Page 50: E. F. G. H. I. - 1 File Download

Age Malnutrition Bone disorders: osteoporosis Systemic disorders: diabetes, Marfan's syndrome and Ehlers-Danlos syndrome cause

abnormal musculoskeletal healing. Drugs: steroids, non steroidal anti inflammatory agents. Type of bone: Cancellous (spongy) bone fractures are usually more stable, involve greater

surface areas, and have a better blood supply than cortical (compact) bone fractures.

Degree of Trauma: The more extensive the injury to bone and surrounding soft tissue, the poorer the outcome.

Vascular Injury: Especially the femoral head, talus, and scaphoid bones. Degree of Immobilization Intra-articular Fractures: These fractures communicate with synovial fluid, which contains

collagenases that retard bone healing. Separation of Bone Ends: Normal apposition of fracture fragments is needed for union to

occur. Inadequate reduction, excessive traction, or interposition of soft tissue will prevent healing.

Infection

Next question

Page 51: E. F. G. H. I. - 1 File Download

Question 23 of 147

Of the options below, which does not cause lymphadenopathy?

Kawasaki disease

Systemic Lupus Erthematosus

Phenytoin

Hydrallazine

Amiodarone

Please rate this question:

Discuss and give feedback

Next question

Lymphadenopathy

Lymphadenopathy in the neck, axillae, groins and abdomen

Need to note: solitary/multiple, defined/indistinct, hard/rubbery/soft, tender/painless

Causes of lymphadenopathy

Mnemonic: Hodgkins disease H aematological: Hodgkins lymphoma, NHL, Leukaemia O ncological: metastases D ermatopathic lympadenitis G aucher's disease K awasaki disease I nfections: TB, glandular fever, Syphilis N iemann Pick disease S erum sickness D rug reaction (phenytoin) I mmunological (SLE) S arcoidosis

Page 52: E. F. G. H. I. - 1 File Download

E ndocrinological (Hyperthyroidism) A ngioimmunoplastic lymphadenopathy S LE E osinophilic granulomatosis

Next question

Page 53: E. F. G. H. I. - 1 File Download

Question 24 of 147

A 23 year old man is reviewed on the ward 10 days following a laparotomy. The wound is inspected and is healing well. Which of the following processes is least likely to be occurring in the wound at this stage?

Angiogenesis

Synthesis of collagen

Necrosis of fibroblasts

Secretion of matrix metalloproteinases by fibroblasts

Proliferation of fibroblasts

Vasculogenesis vs Angiogenesis Vascu is new. Angi is pre Vasculogenesis is new vessels developing in situ from existing mesenchyme. Angiogenesis is vessels develop from sprouting off pre-existing arteries.

Fibroblasts are an important cell type in healing wounds. They typically proliferate in the early phases of wound healing. They release matrix metalloproteinases and these facilitate in the remodelling of the matrix within the healing wound. Necrosis in a healing wound would be unusual as wounds will tend to show clinical evidence of angiognesis by this time. Please rate this question:

Discuss and give feedback

Next question

Wound healing

Surgical wounds are either incisional or excisional and either clean, clean contaminated or dirty. Although the stages of wound healing are broadly similar their contributions will vary according to the wound type. The main stages of wound healing include:

Page 54: E. F. G. H. I. - 1 File Download

Haemostasis

Minutes to hours following injury Vasospasm in adjacent vessels, platelet plug formation and generation of fibrin rich clot.

Inflammation

Typically days 1-5 Neutrophils migrate into wound (function impaired in diabetes). Growth factors released, including basic fibroblast growth factor and vascular endothelial

growth factor.

Fibroblasts replicate within the adjacent matrix and migrate into wound. Macrophages and fibroblasts couple matrix regeneration and clot substitution.

Regeneration

Typically days 7 to 56

Platelet derived growth factor and transformation growth factors stimulate fibroblasts and epithelial cells.

Fibroblasts produce a collagen network. Angiogenesis occurs and wound resembles granulation tissue.

Remodeling

From 6 weeks to 1 year Longest phase of the healing process and may last up to one year (or longer). During this phase fibroblasts become differentiated (myofibroblasts) and these facilitate

wound contraction. Collagen fibres are remodeled. Microvessels regress leaving a pale scar.

The above description represents an idealised scenario. A number of diseases may distort this process. Neovascularisation is an important early process. Endothelial cells may proliferate in the wound bed and recanalise to form a vessel. Vascular disease, shock and sepsis can all compromise microvascular flow and impair healing. Conditions such as jaundice will impair fibroblast synthetic function and immunity with a detrimental effect in most parts of the healing process. Problems with scars:

Hypertrophic scars Excessive amounts of collagen within a scar. Nodules may be present histologically containing randomly arranged fibrils within and parallel fibres on the surface. The tissue itself is confined to the

Page 55: E. F. G. H. I. - 1 File Download

extent of the wound itself and is usually the result of a full thickness dermal injury. They may go on to develop contractures. Image of hypertrophic scarring. Note that it remains confined to the boundaries of the original wound:

Image sourced from Wikipedia

Keloid scars

Excessive amounts of collagen within a scar. Typically a keloid scar will pass beyond the boundaries of the original injury. They do not contain nodules and may occur following even trivial injury. They do not regress over time and may recur following removal. Image of a keloid scar. Note the extension beyond the boundaries of the original incision:

Page 56: E. F. G. H. I. - 1 File Download

Image sourced from Wikipedia

Drugs which impair wound healing:

Non steroidal anti inflammatory drugs

Steroids Immunosupressive agents Anti neoplastic drugs

Closure Delayed primary closure is the anatomically precise closure that is delayed for a few days but before granulation tissue becomes macroscopically evident. Secondary closure refers to either spontaneous closure or to surgical closure after granulation tissue has formed.

Next question

Page 57: E. F. G. H. I. - 1 File Download

Question 25 of 147

A 25 year old women presents with a slowly enlarging mass on the side of the face. Clinical examination demonstrates that the mass is located in the tail of the parotid gland. There is no evidence of facial nerve involvement. What is the most likely cause?

Sialolithiasis

Adenocarcinoma

Warthins tumour

Oncocytoma

Pleomorphic adenoma

Pleomorphic adenomas are the commonest tumours of the parotid gland and are often slow growing, smooth and mobile. Warthins tumours are typically found in elderly males and are composed of multiple cysts and solid components consisting of lymphoid tissue. Warthins tumours are most often found in the tail of the parotid gland, but not in 25 year old females, where a pleomorphic adenoma remains the most likely lesion. Please rate this question:

Discuss and give feedback

Next question

Parotid gland clinical

Benign neoplasms Up to 80% of all salivary gland tumours occur in the parotid gland and up to 80% of these are benign. There is no consistent correlation between the rate of growth and the malignant potential of the lesion. However, benign tumours should not invade structures such as the facial nerve. With the exception of Warthins tumours, they are commoner in women than men. The median age of developing a lesion is in the 5th decade of life. Benign tumour types

Tumour type Features

Page 58: E. F. G. H. I. - 1 File Download

Tumour type Features

Benign pleomorphic adenoma or

benign mixed tumor

Most common parotid neoplasm (80%)

Proliferation of epithelial and myoepithelial cells of the ducts

and an increase in stromal components

Slow growing, lobular, and not well encapsulated

Recurrence rate of 1-5% with appropriate excision

(parotidectomy)

Recurrence possibly secondary to capsular disruption during

surgery

Malignant degeneration occurring in 2-10% of adenomas

observed for long periods, with carcinoma ex-pleomorphic adenoma occurring most frequently as adenocarcinoma

Warthin tumor (papillary

cystadenoma lymphoma or adenolymphoma)

Second most common benign parotid tumor (5%)

Most common bilateral benign neoplasm of the parotid

Marked male as compared to female predominance

Occurs later in life (sixth and seventh decades)

Presents as a lymphocytic infiltrate and cystic epithelial

proliferation

May represent heterotopic salivary gland epithelial tissue

trapped within intraparotid lymph nodes

Incidence of bilaterality and multicentricity of 10%

Malignant transformation rare (almost unheard of)

Monomorphic adenoma Account for less than 5% of tumours

Slow growing

Consist of only one morphological cell type (hence term

mono)

Include; basal cell adenoma, canalicular adenoma, oncocytoma, myoepitheliomas

Haemangioma Should be considered in the differential of a parotid mass in a

child

Accounts for 90% of parotid tumours in children less than 1

year of age

Hypervascular on imaging

Spontaneous regression may occur and malignant transformation is almost unheard of

Malignant salivary gland tumours

Page 59: E. F. G. H. I. - 1 File Download

Types of malignancy

Mucoepidermoid

carcinoma

30% of all parotid malignancies

Usually low potential for local invasiveness and metastasis (depends

mainly on grade)

Adenoid cystic

carcinoma

Unpredictable growth pattern

Tendency for perineural spread

Nerve growth may display skip lesions resulting in incomplete excision

Distant metastasis more common (visceral rather than nodal spread)

5 year survival 35%

Mixed tumours Often a malignancy occurring in a previously benign parotid lesion

Acinic cell carcinoma Intermediate grade malignancy

May show perineural invasion

Low potential for distant metastasis

5 year survival 80%

Adenocarcinoma Develops from secretory portion of gland

Risk of regional nodal and distant metastasis

5 year survival depends upon stage at presentation, may be up to 75% with small lesions with no nodal involvement

Lymphoma Large rubbery lesion, may occur in association with Warthins tumours

Diagnosis should be based on regional nodal biopsy rather than parotid

resection

Treatment is with chemotherapy (and radiotherapy)

Diagnostic evaluation

Plain x-rays may be used to exclude calculi Sialography may be used to delineate ductal anatomy FNAC is used in most cases

Superficial parotidectomy may be either diagnostic of therapeutic depending upon the nature of the lesion

Where malignancy is suspected the primary approach should be definitive resection rather than excisional biopsy

CT/ MRI may be used in cases of malignancy for staging primary disease

Page 60: E. F. G. H. I. - 1 File Download

Treatment

For nearly all lesions this consists of surgical resection, for benign disease this will usually consist of a superficial parotidectomy. For malignant disease a radical or extended radical parotidectomy is performed. The facial nerve is included in the resection if involved. The need for neck dissection is determined by the potential for nodal involvement. Other parotid disorders HIV infection

Lymphoepithelial cysts associated with HIV occur almost exclusively in the parotid

Typically presents as bilateral, multicystic, symmetrical swelling Risk of malignant transformation is low and management usually conservative

Sjogren syndrome

Autoimmune disorder characterised by parotid enlargement, xerostomia and keratoconjunctivitis sicca

90% of cases occur in females Second most common connective tissue disorder Bilateral, non tender enlargement of the gland is usual Histologically, the usual findings are of a lymphocytic infiltrate in acinar units and

epimyoepithelial islands surrounded by lymphoid stroma Treatment is supportive There is an increased risk of subsequent lymphoma

Sarcoid

Parotid involvement occurs in 6% of patients with sarcoid

Bilateral in most cases Gland is not tender Xerostomia may occur Management of isolated parotid disease is usually conservative

Next question

Page 61: E. F. G. H. I. - 1 File Download

Question 26 of 147

Beta-naphthalamine is associated with which of the following malignancies?

Lung cancer

Bowel cancer

Bladder cancer

Liver cancer

Renal cancer

Beta-naphthalamine is used in the rubber industry. The following factors are associated with the development of bladder cancer:

smoking occupational: aniline dyes used in printing and textile industry, rubber manufacture

schistosomiasis drugs: cyclophosphamide

Please rate this question:

Discuss and give feedback

Next question

Occupational cancers

Occupational cancers accounted for 5.3% cancer deaths in 2005. In men the main cancers include:

Mesothelioma Bladder cancer

Non melanoma skin cancer Lung cancer

Page 62: E. F. G. H. I. - 1 File Download

Sino nasal cancer

Occupations with high levels of occupational tumours include:

Construction industry

Working with coal tar and pitch Mining Metalworkers Working with asbestos (accounts for 98% of all mesotheliomas)

Working in rubber industry

Shift work has been linked to breast cancer in women (Health and safety executive report RR595). The latency between exposure and disease is typically 15 years for solid tumours and 20 for leukaemia. Many occupational cancers are otherwise rare. For example sino nasal cancer is an uncommon tumour, 50% will be SCC. They are linked to conditions such as wood dust exposure and unlike lung cancer is not strongly linked to cigarette smoking. Another typical occupational tumour is angiosarcoma of the liver which is linked to working with vinyl chloride. Again in the non occupational context this is an extremely rare sporadic tumour.

Next question

Page 63: E. F. G. H. I. - 1 File Download

Question 27 of 147

A 56 year old man with Wilsons disease presents with right upper quadrant discomfort. An ultrasound scan is performed and this demonstrates a mass lesion in the right lobe of the liver. What is the most appropriate method of establishing the underlying diagnosis?

PET CT scan

Ultrasound guided biopsy

Measurement of serum alpha feto protein

MRI scan of the liver

CT scan of the liver

High AFP + chronic liver inflammation = Hepatocellular carcinoma.

This is likely to be a hepatocellulcar carcinoma. Diagnosis is usually made by AFP measurement (with further imaging depending on the result). Biopsy should not be performed as it may seed the tumour. Chronic liver diseases such as Wilsons disease (Hepato-lenticular degeneration) increase the risk. Please rate this question:

Discuss and give feedback

Next question

Liver tumours

Primary liver tumours

The most common primary tumours are cholangiocarcinoma and hepatocellular carcinoma. Overall metastatic disease accounts for 95% of all liver malignancies making the primary liver tumours comparatively rare. Primary liver tumours include:

Cholangiocarcinoma

Hepatocellular carcinoma Hepatoblastoma

Page 64: E. F. G. H. I. - 1 File Download

Sarcomas (Rare) Lymphomas Carcinoids (most often secondary although primary may occur)

Hepatocellular carcinoma These account for the bulk of primary liver tumours (75% cases). Its worldwide incidence reflects its propensity to occur on a background of chronic inflammatory activity. Most cases arise in cirrhotic livers or those with chronic hepatitis B infection, especially where viral replication is actively occurring. In the UK it accounts for less than 5% of all cancers, although in parts of Asia its incidence is 100 per 100,000. The majority of patients (80%) present with existing liver cirrhosis, with a mass discovered on screening ultrasound. Diagnosis

CT/ MRI (usually both) are the imaging modalities of choice a-fetoprotein is elevated in almost all cases Biopsy should be avoided as it seeds tumours cells through a resection plane. In cases of diagnostic doubt serial CT and αFP measurements are the preferred strategy.

Treatment

Patients should be staged with liver MRI and chest, abdomen and pelvic CT scan. The testis should be examined in males (testicular tumours may cause raised AFP). PET CT

may be used to identify occult nodal disease.

Surgical resection is the mainstay of treatment in operable cases. In patients with a small primary tumour in a cirrhotic liver whose primary disease process is controlled, consideration may be given to primary whole liver resection and transplantation.

Liver resections are an option but since most cases occur in an already diseased liver the operative risks and post-operative hepatic dysfunction are far greater than is seen following metastectomy.

These tumours are not particularly chemo or radiosensitive however, both may be used in a palliative setting. Tumour ablation is a more popular strategy.

Survival

Poor, overall survival is 15% at 5 years. Cholangiocarcinoma

This is the second most common type of primary liver malignancy. As its name suggests these tumours arise in the bile ducts. Up to 80% of tumours arise in the extra hepatic biliary tree. Most patients present with jaundice and by this stage the majority will have disease that is not resectable. Primary sclerosing cholangitis is the main risk factor. In deprived countries typhoid and liver flukes are also major risk factors. Diagnosis

Page 65: E. F. G. H. I. - 1 File Download

Patients will typically have an obstructive picture on liver function tests. CA 19-9, CEA and CA 125 are often elevated CT/ MRI and MRCP are the imaging methods of choice.

Treatment

Surgical resection offers the best chance of cure. Local invasion of peri hilar tumours is a particular problem and this coupled with lobar atrophy will often contra indicate surgical resection.

Palliation of jaundice is important, although metallic stents should be avoided in those considered for resection.

Survival Is poor, approximately 5-10% 5 year survival.

Next question

Page 66: E. F. G. H. I. - 1 File Download

Question 28 of 147

A 55 year old man has suffered from reflux oesophagitis for many years. During a recent endoscopy a biopsy is taken from the distal oesophagus. The histopathology report indicates that cells are identified with features of coarse chromatin and abnormal mitoses. The cells are confined to the superficial epithelial layer only. Which of the following accounts for this process?

Metaplasia

Apoptosis

Autoimmune oesophagitis

Dysplasia

Infection with Helicobacter pylori

Dysplasia = pre cancerous

Dysplasia tends to develop as a result of prolonged stimulation by precipitants. Removal of these precipitants may possibly reverse these changes. Replacement of differentiated cells with another cell type describes metaplasia rather than dysplasia. The absence of invasion distinguishes this from malignancy. Please rate this question:

Discuss and give feedback

Next question

Dysplasia

Premalignant condition Disordered growth and differentiation of cells Alteration in size, shape, and organization of cells Features increased abnormal cell growth (increased number of mitoses/abnormal mitoses

and cellular differentiation)

Underlying connective tissue is not invaded Causes include smoking, Helicobacter pylori, Human papilloma virus Main differences to metaplasia is that dysplasia is considered to be part of carcinogenesis

(pre cancerous) and is associated with a delay in maturation of cells rather than differentiated cells replacing one another

Page 67: E. F. G. H. I. - 1 File Download

The absence of invasion differentiates dysplasia from invasive malignancy Severe dysplasia with foci of invasion are well recognised

Next question

Page 68: E. F. G. H. I. - 1 File Download

Question 29 of 147

Which one of the following confers the least risk of developing osteoporosis?

Obesity

Long term unfractionated heparin therapy

Gastrectomy

Osteogenesis imperfecta

Diabetes

Low body weight is a risk factor for osteoporosis. Please rate this question:

Discuss and give feedback

Next question

Osteoporosis

Risk factors

Family history

Female sex Increasing age Deficient diet Sedentary lifestyle

Smoking Premature menopause Low body weight Caucasians and Asians

Diseases which predispose

Page 69: E. F. G. H. I. - 1 File Download

Endocrine: glucocorticoid excess (e.g. Cushing's, steroid therapy), hyperthyroidism, hypogonadism (e.g. Turner's, testosterone deficiency), growth hormone deficiency, hyperparathyroidism, diabetes mellitus

Multiple myeloma, lymphoma Gastrointestinal problems: inflammatory bowel disease, malabsorption (e.g. Coeliacs),

gastrectomy, liver disease Rheumatoid arthritis

Long term heparin therapy Chronic renal failure Osteogenesis imperfecta, homocystinuria

Next question

Page 70: E. F. G. H. I. - 1 File Download

Question 30 of 147

A 63 year old man has a history of claudication that has been present for many years. He is recently

evaluated in the clinic and a duplex scan shows that he has an 85% stenosis of the superficial

femoral artery. Two weeks later he presents with a 1 hour history of severe pain in his leg. On

examination he has absent pulses in the affected limb and it is much cooler than the contra-lateral

limb. Which process best accounts for this presentation?

Thrombosis

Embolus

Atheroma growth

Sub intimal dissection

Anaemia

Theme from April 2012 Exam

In an existing lesion a complication such as thrombosis is more likely than embolus. These patients

should receive heparin and imaging with duplex scanning. Whilst an early surgical bypass or intra-

arterial thrombolysis may be indicated, an embolectomy should not generally be performed as the

lesion is not an embolus and the operation therefore ineffective.

Please rate this question:

Discuss and give feedback

Next question

Claudication

Page 71: E. F. G. H. I. - 1 File Download

Claudication is a condition in which patients develop pain in a limb during periods of exercise. The

underlying disorder is usually that of arterial insufficiency. Atheroma develops in the arterial wall and

once this occludes >50-75% of the lumenal diameter the supply to metabolising tissues distally may

become compromised. The typical claudicant complains of calf pain that is worse on exercise and

relieved by rest. This typical description assumes that the SFA is the site of disease, more proximal

disease may present with other symptoms such as buttock claudication and impotence.

The history is usually a progressive one, patients presenting as an emergency with severe pain,

diminished sensation, pallor and absent pulses have critical limb ischaemia. This may complicate

claudication and usually indicates a plaque related complication, such as thrombosis.

Risk factors

Risk factors for claudication include smoking, diabetes and hyperlipidaemia.

Diagnosis

Diagnostic work -up includes measurement of ankle- brachial pressure indices, duplex scanning and

formal angiography.

Treatment

Those with long claudication distances, no ulceration or gangrene may be managed conservatively.

Patients with rest pain, ulceration or gangrene will almost always require intervention. All patients

should receive an antiplatelet agent and a statin unless there are compelling contra-indications.

Next question

Page 72: E. F. G. H. I. - 1 File Download

Question 31 of 147

The following are true of carcinoid tumours except:

When present in the appendix tip and measure less than 2 cm have an excellent prognosis

Even when metastatic disease is present it tends to follow a protracted course

When present in the appendix body tend to present with carcinoid syndrome even when

liver metastases are not present

May be imaged using 5 HIAA radionucleotide scanning

Advanced appendiceal carcinoids may require right hemicolectomy

Rule of thirds: 1/3 multiple 1/3 small bowel 1/3 metastasize 1/3 second tumour

Liver metastases are necessary for the presence of carcinoid syndrome. Please rate this question:

Discuss and give feedback

Next question

Carcinoid syndrome

Carcinoid tumours secrete serotonin Originate in neuroendocrine cells mainly in the intestine (midgut-distal ileum/appendix)

Can occur in the rectum, bronchi Hormonal symptoms mainly occur when disease spreads outside the bowel

Clinical features

Onset: insidious over many years

Page 73: E. F. G. H. I. - 1 File Download

Flushing face Palpitations Pulmonary valve stenosis and tricuspid regurgitation causing dyspnoea Asthma

Severe diarrhoea (secretory, persists despite fasting)

Investigation

5-HIAA in a 24-hour urine collection

Somatostatin receptor scintigraphy CT scan Blood testing for chromogranin A

Treatment

Octreotide

Surgical removal

Next question

Page 74: E. F. G. H. I. - 1 File Download

Question 32 of 147

During a difficult femoro-popliteal bypass operation the surgeon inadvertently places a clamp across the femoral nerve. It remains there for most of the procedure. At the end of the operation the nerve is inspected, it is in continuity but has evidence of being crushed. Which of the following is most likely to occur over the following months?

Wallerian degeneration

Rapid restoration of neuronal function because the axon itself is intact

Normal but delayed neuronal transmission due to disruption of the myelin

Absence of neuroma formation

None of the above

A neuronal injury such as this will result in Wallerian degeneration even though the nerve remains in continuity. Neuromas may well form. Please rate this question:

Discuss and give feedback

Next question

Nerve injury

There are 3 types of nerve injury:

Neuropraxia Nerve intact but electrical conduction is affected

Full recovery

Autonomic function preserved

Wallerian degeneration does not occur

Axonotmesis Axon is damaged and the myelin sheath is preserved. The connective tissue

framework is not affected.

Wallerian degeneration occurs.

Page 75: E. F. G. H. I. - 1 File Download

Neurotmesis Disruption of the axon, myelin sheath and surrounding connective tissue.

Wallerian degeneration occurs.

Wallerian Degeneration

Axonal degeneration distal to the site of injury.

Typically begins 24-36 hours following injury. Axons are excitable prior to degeneration occurring. Myelin sheath degenerates and is phagocytosed by tissue macrophages.

Nerve repair

Neuronal repair may only occur physiologically where nerves are in direct contact. Where a large defect is present, the process of nerve regeneration is hampered. It may not occur at all or result in the formation of a neuroma. Where nerve regrowth occurs it is typically at a rate of 1mm per day.

Next question

Page 76: E. F. G. H. I. - 1 File Download

Question 33 of 147

A 38 year old lady who smokes heavily presents with recurrent episodes of infection in the right

breast. On examination she has an indurated area at the lateral aspect of the nipple areaolar

complex. Imaging shows no mass lesions. What is the most likely diagnosis?

Duct ectasia

Periductal mastitis

Pagets disease of the nipple

Mondors disease of the breast

Radial scar

Periductal mastitis is common in smokers and may present with recurrent infections. Treatment is

with co-amoxyclav. Mondors disease of the breast is a localised thrombophlebitis of a breast vein.

Please rate this question:

Discuss and give feedback

Next question

Duct ectasia

Duct ectasia is a dilatation and shortening of the terminal breast ducts within 3cm of the nipple. It is

common and the incidence increases with age. It typically presents with nipple retraction and

occasionally creamy nipple discharge. It may be confused with periductal mastitis, which presents in

younger women, the vast majority of which are smokers. Periductal mastitis typically presents with

periareolar or sub areolar infections and may be recurrent.

Page 77: E. F. G. H. I. - 1 File Download

Patients with troublesome nipple discharge may be treated by microdochectomy (if young) or total

duct excision (if older).

Next question

Page 78: E. F. G. H. I. - 1 File Download

Question 34 of 147

Which of the following statements relating to chronic inflammation is true?

Chronic inflammation is mainly secondary to acute inflammation

Neutrophils are the predominant cells involved

Growth factors are not involved in the process

Appendicitis is mainly a form of chronic inflammation

Fibrosis is a macroscopic feature

Macroscopic features include:

Ulcers

Fibrosis Granulomatous process

It most commonly occurs as a primary event rather than as a result of acute inflammation. Please rate this question:

Discuss and give feedback

Next question

Chronic inflammation

Overview Chronic inflammation may occur secondary to acute inflammation.In most cases chronic inflammation occurs as a primary process. These may be broadly viewed as being one of three main processes:

Persisting infection with certain organisms such as Mycobacterium tuberculosis which results in delayed type hypersensitivity reactions and inflammation.

Prolonged exposure to non-biodegradable substances such as silica or suture materials which may induce an inflammatory response.

Page 79: E. F. G. H. I. - 1 File Download

Autoimmune conditions involving antibodies formed against host antigens.

Acute vs. Chronic inflammation

Acute inflammation Chronic inflammation

Changes to existing vascular structure and increased

permeability of endothelial cells

Angiogenesis predominates

Infiltration of neutrophils Macrophages, plasma cells and lymphocytes predominate

Process may resolve with:

Suppuration

Complete resolution

Abscess formation

Progression to chronic inflammation

Healing by fibrosis

Healing by fibrosis is the main result

Granulomatous inflammation

A granuloma consists of a microscopic aggregation of macrophages (with epithelial type arrangement =epitheliod). Large giant cells may be found at the periphery of granulomas. Mediators Growth factors released by activated macrophages include agents such as interferon and fibroblast growth factor (plus many more). Some of these such as interferons may have systemic features resulting in systemic symptoms and signs, which may be present in individuals with long standing chronic inflammation. The finding of granulomas is pathognomonic of chronic inflammation, as illustrated in this biopsy

from a patient with colonic Crohns disease

Page 81: E. F. G. H. I. - 1 File Download

1 / 3

Question 35-37 of 147

Theme: Paediatric ano-rectal disorders

A. Ulcerative colitis

B. Juvenile polyps

C. Haemorroids

D. Intussceception

E. Rectal cancer

F. Anal fissure

G. Arteriovenous malformation

Please select the most likely cause for the condition described. Each option may be used once,

more than once or not at all.

35. A 4 year old boy is brought to the clinic. He gives a history of difficult, painful defecation with

bright red rectal bleeding.

You answered Ulcerative colitis

The correct answer is Anal fissure

Theme from April 2012 Exam

Painful rectal bleeding in this age group is typically due to a fissure. Treatment should include

stool softeners and lifestyle advice.

36. A 2 year old has a history of rectal bleeding. The parents notice that post defecation, a cherry red

lesion is present at the anal verge.

You answered Ulcerative colitis

Page 82: E. F. G. H. I. - 1 File Download

The correct answer is Juvenile polyps

Theme from September 2012 Exam

These lesions are usually hamartomas and this accounts for the colour of the lesions. Although

the lesions are not themselves malignant they serve as a marker of an underlying polyposis

disorder.

37. A 12 year old is brought to the colorectal clinic with a history of rectal bleeding, altered bowel

habit, weight loss and malaise. Abdominal examination is normal.

Ulcerative colitis

The systemic features in the history are strongly suggestive of inflammatory bowel disease rather

than the other causes.

Please rate this question:

Discuss and give feedback

Next question

Paediatric proctology

Children may present with altered bowel habit and/ or rectal bleeding. Classical haemorroidal

disease is relatively rare in children. Painful bright red rectal bleeding is much more common since

constipation is a relatively common childhood disorder. The hard stool causes a tear of the ano-

rectal mucosa with subsequent fissure. The pain from the fissure must be addressed promptly or the

child will delay defecation and this fissure will worsen.

Inflammatory bowel disease may present in a similar pattern in paediatric practice with altered bowel

habit (usually diarrhoea) and bleeding. Systemic features may be present and investigation with an

endoscopy may be required.

Children with intussceception usually present at a relatively young age and the history is usually one

of colicky abdominal pain, together with a mass on clinical examination.The often cited red current

jelly type stool is a rare but classical feature.

Page 83: E. F. G. H. I. - 1 File Download

Juvenile polyps may occur as part of the familial polyposis coli syndromes. The lesions, which are

hamartomas, are often cherry red if they protrude externally.

Next question

Page 84: E. F. G. H. I. - 1 File Download

Question 38 of 147

You review a 42-year-old woman 8 months following a renal transplant for focal segmental glomerulosclerosis. She is on a combination of tacrolimus, mycophenolate, and prednisolone. She has now presented with a five day history of feeling generally unwell with jaundice, fatigue and arthralgia. On examination she has jaundice, widespread lymphadenopathy and hepatomegaly. What is the most likely diagnosis?

Hepatitis C

Epstein-Barr virus

HIV

Hepatitis B

Cytomegalovirus

Post transplant complications CMV: 4 weeks to 6 months post transplant EBV: post transplant lymphoproliferative disease. > 6 months post transplant

Post transplant lymphoproliferative disorder is most commonly associated with Epstein-Barr virus. It typically occurs 6 months post transplant and is associated with high dose immunosupressant therapy. Remember cytomegalovirus presents within the first 4 weeks to 6 months post transplant. Please rate this question:

Discuss and give feedback

Next question

Renal transplant:HLA typing and graft failure

The human leucocyte antigen (HLA) system is the name given to the major histocompatibility complex (MHC) in humans. It is coded for on chromosome 6. Some basic points on the HLA system

Page 85: E. F. G. H. I. - 1 File Download

Class 1 antigens include A, B and C. Class 2 antigens include DP,DQ and DR When HLA matching for a renal transplant the relative importance of the HLA antigens are as

follows DR > B > A

Graft survival

1 year = 90%, 10 years = 60% for cadaveric transplants 1 year = 95%, 10 years = 70% for living-donor transplants

Post-op problems

ATN of graft Vascular thrombosis Urine leakage

UTI

Hyperacute acute rejection

Due to antibodies against donor HLA type 1 antigens

Rarely seen due to HLA matching

Acute graft failure (< 6 months)

Usually due to mismatched HLA Other causes include cytomegalovirus infection

Management: give steroids, if resistant use monoclonal antibodies

Causes of chronic graft failure (> 6 months)

Chronic allograft nephropathy

Ureteric obstruction Recurrence of original renal disease (MCGN > IgA > FSGS)

Next question

Page 86: E. F. G. H. I. - 1 File Download

Question 39 of 147

Which of the following processes facilitates phagocytosis?

Apoptosis

Opsonisation

Proteolysis

Angiogenesis

Necrosis

Theme from September 2014 Exam Opsonisation will facilitate phagocytosis. The micro-organism becomes coated with antibody, C3b and certain acute phase proteins. The macrophages and neutrophils have up regulation of phagocytic cell surface receptors in these circumstances, a process mediated by pro-inflammatory cytokines. These cells then engulf the micro organism. Please rate this question:

Discuss and give feedback

Next question

Phagocytosis

Ingestion of pathogens or foreign materials by cells First step is opsonisation whereby the organism is coated by antibody Second step is adhesion to cell surface Third step is pseudopodial extension to form a phagocytic vacuole

Lysosomes fuse with vacuole and degrade contents

Next question

Page 87: E. F. G. H. I. - 1 File Download

Question 40 of 147

Features which are evaluated for the grading of breast cancer include all the following, except:

Tubule formation

Mitoses

Nuclear pleomorphism

Tumour necrosis

Coarse chromatin

The necrosis of a tumour may be suggestive of a high grade tumour which has out grown its blood

supply. However, the grading of breast cancer which classically follows the Bloom -Richardson

grading model will tend to favor nuclear appearances (which include mitoses, coarse chromatin and

pleomorphism). Tubule formation is an important marker of the degree of differentiation with

formation of tubular structures being associated with well differentiated tumours.

Please rate this question:

Discuss and give feedback

Next question

Tumour grading

Tumours may be graded according to their degree of differentiation, mitotic activity and other

features. Grade 1 tumours are the most differentiated and grade 3 or 4 the least. The assessment is

subjective, in most cases high grade equates to poor prognosis, or at least rapid growth.

Tumours of glandular epithelium will tend to arrange themselves into acinar type structures

Page 88: E. F. G. H. I. - 1 File Download

containing a central lumen. Well differentiated tumours may show excellent acinar formation and

poorly differentiated tumours simply clumps of cells around a desmoplastic stroma. Sometimes

tumours demonstrate mucous production without evidence of acinar formation. Since mucous

production is evidence of a glandular function such tumours are often termed mucinous

adenocarcinoma.

Squamous cell tumours will typically produce structures resembling epithelial cell components. Well

differentiated tumours may also produce keratin (depending upon tissue of origin).

Next question

Page 89: E. F. G. H. I. - 1 File Download

Question 41 of 147

A 34 year old man is diagnosed with an aggressive caecal adenocarcinoma. His sister died from the

same disease at 38 years of age. His mother died from endometrial cancer at the age of 41. What is

the most likely underlying abnormality?

Familial adenopolypomatosis coli

Gardeners syndrome

Mutation of mismatch repair genes

Deletion of chromosome 6

MYH gene mutation

Lynch syndrome which is characterised by aggressive right sided colonic malignancy and

endometrial cancer is caused by microsatellite instability of DNA repair genes.

Please rate this question:

Discuss and give feedback

Next question

Genetics of colorectal cancer

The lifetime risk of colorectal cancer in the UK population is 5%. Up to 5% of newly diagnosed bowel

cancers will be in those individuals who have a high genetically acquired risk of bowel cancer.

Cancers arising in the low-moderate genetic risk group comprise approximately 30% of newly

diagnosed bowel cancer.

Page 90: E. F. G. H. I. - 1 File Download

Genetics of inherited colorectal cancer syndromes

Syndrome Features Genes implicated

FAP More than 100 adenomatous polyps affecting the colon and

rectum. Duodenal and fundic glandular polyps

APC (over 90%)

Gardner

syndrome

As FAP but with desmoid tumours and mandibular osteomas APC

Turcots syndrome Polyposis and colonic tumours and CNS tumours APC +MLH1 and

PMS2

HNPCC Colorectal cancer without extensive polyposis. Endometrial

cancer, renal and CNS

MSH2, MLH1, PMS2

and GTBP

Peutz-Jeghers

syndrome

Hamartomatous polyps in GI tract and increased risk of GI

malignancy

LKB1 andSTK11 (in

up to 70%)

Cowden disease Multiple hamartomas (see below) PTEN (85%)

MYH associated

polyposis

Autosomal recessive, multiple adenomatous polyps in GI tract,

those in colon having somatic KRAS mutations

MYH

FAP

Autosomal dominant condition, affects 1 in 12,000. Accounts for 0.5% of all CRCs. Lifetime

incidence of colorectal cancer in untreated FAP =100%. Up to 25% cases are caused by de-novo

germ line mutations and show no prior family history. The APC tumour suppressor gene is affected

in most cases.

APC in non inherited colorectal cancer

Up to 80% of sporadic colorectal cancers will have somatic mutations that inactivate APC[1]. Both

alleles are usually affected. Although the APC protein more than likely has multiple critical cellular

functions, the best-established role for APC in the cancer process is as a major binding partner and

regulator of the β- catenin protein in the so-called canonical or β- catenin dependent Wnt signaling

Page 91: E. F. G. H. I. - 1 File Download

pathway.

HNPCC (Lynch syndrome)

HNPCC cancers differ from conventional tumours in a number of respects. In the colon the tumours

are more likely to be right sided, histologically they are more likely to be mucinous and have dense

lymphocytic infiltrates. To be diagnosed as having HNPCC individuals must show typically HNPCC

tumours in at least three individuals, (one of whom must be a first degree relative to the other two).

In at least two successive generations. At least one cancer must be diagnosed under the age of 50.

FAP must be excluded and tumours should be verified by pathological identification (Amsterdam

criteria). The genetic changes in HNPCC stem primarily from microsatellite instability affecting DNA

mismatch repair genes. In HNPCC the mismatch repair genes most commonly implicated include;

MSH2 and MLH1 and these occur in up to 70% of people with HNPCC. The finding of microsatellite

instability is unusual in sporadic colorectal cancers. Approximately 60% of individuals who fulfill the

Amsterdam criteria will not be found to have evidence of mismatch repair gene defects on genetic

testing. The risk of developing colorectal cancer in those who have not demonstrated mutation of the

mis match repair genes is increased if they fulfill the Amsterdam criteria, but not

the extent that it is increased in those who fulfill the criteria AND have evidence of mis match repair

gene defects.

KRAS Mutations

The RAS family of small G proteins act as molecular switches downstream of growth factor

receptors. KRAS and the other two members of the family; HRAS and NRAS, are the site of

mutation in approximately 40% of colorectal cancers. When adenomas are examined the proportion

of adenomas less than 1cm showing KRAS mutations was only 10% which contrasts with 50% in

those lesions greater than 1cm.

p53 mutations

The p53 protein functions as a key transcriptional regulator of genes that encode proteins with

functions in cell-cycle checkpoints at the G1/S and G2/M boundaries, in promoting apoptosis, and in

restricting angiogenesis . As such, selection for p53 defects at the adenoma-carcinoma transition

may reflect the fact that stresses on tumor cells activate cell-cycle arrest, apoptotic, and

antiangiogenic pathways in cells with wild-type p53 function. Many colonic tumours will demonstrate

changes in the p53 gene that may facilitate tumour progression through from adenoma to

carcinoma.

Cowden syndrome

Also known as multiple hamartoma syndrome. Rare autosomal dominant condition with incidence of

1 in 200,000.. It is characterised by multiple mucocutaneous lesions, trichilemmomas, oral

papillomas and acral keratosis. Most often diagnosed in third decade of life. Breast carcinoma may

occur in up to 50% of patients and conditions such as fibrocystic disease of the breast may occur in

75% of women. Thyroid disease occurs in 75% and may include malignancy. Endoscopic screening

will identify disease in up to 85% although the small bowel is rarely involved. There is a 15-20% risk

of developing colorectal cancer and regular colonoscopic screening from age 45 is recommended.

Terminology

Page 92: E. F. G. H. I. - 1 File Download

Oncogene Oncogenes are genes which have the potential to induce cellular proliferation and avoid

apoptosis. Oncogene mutations are general gain of function and are therefore

dominant. Increased expression of oncogenes are found in most tumours

Tumour

suppressor

gene

These genes generally inhibit cellular proliferation or induce apoptosis. Mutations in

tumour suppressor genes are generally loss of function mutations, and are therefore

recessive. Mutations in both tumour suppressor gene alleles allow cells to proliferate

without restraint

References

1. Fearon, E.R. and B. Vogelstein, A genetic model for colorectal tumorigenesis. Cell, 1990. 61(5): p.

759-67.

Next question

Page 93: E. F. G. H. I. - 1 File Download

Question 42 of 147

Which of the following is associated with poor wound healing?

Jaundice

Patients taking carbamazepine

General anaesthesia using thiopentone

General anaesthesia using ketamine

Multiple sclerosis

Mnemonic to remember factors affecting wound healing: DID NOT HEAL D iabetes I nfection, irradiation D rugs eg steroids, chemotherapy N utritional deficiencies (vitamin A, C & zinc, manganese), Neoplasia O bject (foreign material) T issue necrosis H ypoxia E xcess tension on wound A nother wound L ow temperature, Liver jaundice

Multiple sclerosis is associated with pressure sores, however the cellular healing process is not affected. Please rate this question:

Discuss and give feedback

Next question

Wound healing

Surgical wounds are either incisional or excisional and either clean, clean contaminated or dirty. Although the stages of wound healing are broadly similar their contributions will vary according to the

Page 94: E. F. G. H. I. - 1 File Download

wound type. The main stages of wound healing include: Haemostasis

Minutes to hours following injury Vasospasm in adjacent vessels, platelet plug formation and generation of fibrin rich clot.

Inflammation

Typically days 1-5 Neutrophils migrate into wound (function impaired in diabetes). Growth factors released, including basic fibroblast growth factor and vascular endothelial

growth factor. Fibroblasts replicate within the adjacent matrix and migrate into wound. Macrophages and fibroblasts couple matrix regeneration and clot substitution.

Regeneration

Typically days 7 to 56

Platelet derived growth factor and transformation growth factors stimulate fibroblasts and epithelial cells.

Fibroblasts produce a collagen network.

Angiogenesis occurs and wound resembles granulation tissue.

Remodeling

From 6 weeks to 1 year

Longest phase of the healing process and may last up to one year (or longer). During this phase fibroblasts become differentiated (myofibroblasts) and these facilitate

wound contraction. Collagen fibres are remodeled. Microvessels regress leaving a pale scar.

The above description represents an idealised scenario. A number of diseases may distort this process. Neovascularisation is an important early process. Endothelial cells may proliferate in the wound bed and recanalise to form a vessel. Vascular disease, shock and sepsis can all compromise microvascular flow and impair healing. Conditions such as jaundice will impair fibroblast synthetic function and immunity with a detrimental effect in most parts of the healing process. Problems with scars:

Page 95: E. F. G. H. I. - 1 File Download

Hypertrophic scars

Excessive amounts of collagen within a scar. Nodules may be present histologically containing randomly arranged fibrils within and parallel fibres on the surface. The tissue itself is confined to the extent of the wound itself and is usually the result of a full thickness dermal injury. They may go on to develop contractures. Image of hypertrophic scarring. Note that it remains confined to the boundaries of the original wound:

Image sourced from Wikipedia

Keloid scars Excessive amounts of collagen within a scar. Typically a keloid scar will pass beyond the boundaries of the original injury. They do not contain nodules and may occur following even trivial injury. They do not regress over time and may recur following removal. Image of a keloid scar. Note the extension beyond the boundaries of the original incision:

Page 96: E. F. G. H. I. - 1 File Download

Image sourced from Wikipedia

Drugs which impair wound healing:

Non steroidal anti inflammatory drugs

Steroids Immunosupressive agents Anti neoplastic drugs

Closure Delayed primary closure is the anatomically precise closure that is delayed for a few days but before granulation tissue becomes macroscopically evident. Secondary closure refers to either spontaneous closure or to surgical closure after granulation tissue has formed.

Next question

Page 97: E. F. G. H. I. - 1 File Download

Question 43 of 147

A 55 year old man undergoes a colonoscopy and a colonic polyp is identified. It has a lobular

appearance and is located on a stalk in the sigmoid colon. Which of the processes below best

accounts for this disease?

Apoptosis

Metaplasia

Dysplasia

Calcification

Degeneration

Theme from April 2012 Exam

Most colonic polyps described above are adenomas. These may have associated dysplasia. The

more high grade the dysplasia the greater the level of clinical concern.

Please rate this question:

Discuss and give feedback

Next question

Colonic polyps

Colonic Polyps

May occur in isolation, or greater numbers as part of the polyposis syndromes. In FAP greater than

100 polyps are typically present. The risk of malignancy in association with adenomas is related to

size, and is the order of 10% in a 1cm adenoma. Isolated adenomas seldom give risk of symptoms

Page 98: E. F. G. H. I. - 1 File Download

(unless large and distal). Distally sited villous lesions may produce mucous and if very large,

electrolyte disturbances may occur.

Follow up of colonic polyps

Group Features Action

Low risk 1 or 2 adenomas less than 1cm No follow up or re-colonoscopy

at 5 years

Moderate

risk

3 or 4 small adenomas or 1 adenoma greater than 1cm Re-scope at 3 years

High risk More than 5 small adenomas or more than 3 with 1 of

them greater than 1cm

Re scope at 1 year

From Atkins and Saunders Gut 2002 51 (suppl V:V6-V9). It is important to stratify patients

appropriately and ensure that a complete colonoscopy with good views was performed.

Segmental resection or complete colectomy should be considered when:

1. Incomplete excision of malignant polyp

2. Malignant sessile polyp

3. Malignant pedunculated polyp with submucosal invasion

4. Polyps with poorly differentiated carcinoma

5. Familial polyposis coli

-Screening from teenager up to 40 years by 2 yearly sigmoidoscopy/colonoscopy

-Panproctocolectomy and Ileostomy or Restorative Panproctocolectomy.

Rectal polypoidal lesions may be amenable to trans anal endoscopic microsurgery.

References

Cairns S et al. Guidelines for colorectal cancer screening and surveillance in moderate and high risk

groups (update from 2002). Gut 2010;59:666-690.

Next question

Page 99: E. F. G. H. I. - 1 File Download

Question 44 of 147

A 56 year old lady has just undergone a colonoscopy and a 1.5cm lesion was identified in the

caecum. The histology report states that biopsies have been taken from a sessile serrated polyp with

traditional features. What is the best management option?

Perform a right hemicolectomy

List the patient for colonoscopic polypectomy

Discharge the patient

Re scope the patient in 6 months

Re scope the patient at 3 years

These polyps represent an alternative pathway to progression to carcinoma and may be

diagnostically confused with hyperplastic polyps. Hyperplastic polyps are more common in the left

colon and confer no increased risk. SSA's are more common in the right colon and are usually

larger. Those with "traditional features" on histology have dysplasia with increased risk of malignant

transformation.

Please rate this question:

Discuss and give feedback

Next question

Colonic polyps

Colonic Polyps

May occur in isolation, or greater numbers as part of the polyposis syndromes. In FAP greater than

Page 100: E. F. G. H. I. - 1 File Download

100 polyps are typically present. The risk of malignancy in association with adenomas is related to

size, and is the order of 10% in a 1cm adenoma. Isolated adenomas seldom give risk of symptoms

(unless large and distal). Distally sited villous lesions may produce mucous and if very large,

electrolyte disturbances may occur.

Follow up of colonic polyps

Group Features Action

Low risk 1 or 2 adenomas less than 1cm No follow up or re-colonoscopy

at 5 years

Moderate

risk

3 or 4 small adenomas or 1 adenoma greater than 1cm Re-scope at 3 years

High risk More than 5 small adenomas or more than 3 with 1 of

them greater than 1cm

Re scope at 1 year

From Atkins and Saunders Gut 2002 51 (suppl V:V6-V9). It is important to stratify patients

appropriately and ensure that a complete colonoscopy with good views was performed.

Segmental resection or complete colectomy should be considered when:

1. Incomplete excision of malignant polyp

2. Malignant sessile polyp

3. Malignant pedunculated polyp with submucosal invasion

4. Polyps with poorly differentiated carcinoma

5. Familial polyposis coli

-Screening from teenager up to 40 years by 2 yearly sigmoidoscopy/colonoscopy

-Panproctocolectomy and Ileostomy or Restorative Panproctocolectomy.

Rectal polypoidal lesions may be amenable to trans anal endoscopic microsurgery.

References

Cairns S et al. Guidelines for colorectal cancer screening and surveillance in moderate and high risk

groups (update from 2002). Gut 2010;59:666-690.

Next question

Page 101: E. F. G. H. I. - 1 File Download

Question 45 of 147

A 30 year old male presents with gynaecomastia. Clinically, he is noted to have a nodule in the left testis. What is the most likely diagnosis?

Oestrogen abuse

Seminoma with syncytiotrophoblast giant cells

Teratoma

Choriocarcinoma

Leydig cell tumour

Leydig cell tumours are rare testicular sex cord stromal tumours (which also include sertoli cell tumours) which are associated with hormonal activity.

Patients with Leydig cell tumours may present with gynaecomastia before they notice testicular enlargement.

Majority are benign Histology: eosinophilic cells in columns

Please rate this question:

Discuss and give feedback

Next question

Testicular disorders

Testicular cancer Testicular cancer is the most common malignancy in men aged 20-30 years. Around 95% of cases of testicular cancer are germ-cell tumours. Germ cell tumours may essentially be divided into:

Tumour type Key features

Tumour

markers Pathology

Page 102: E. F. G. H. I. - 1 File Download

Tumour type Key features

Tumour

markers Pathology

Seminoma Commonest

subtype (50%)

Average age at

diagnosis = 40

Even advanced

disease associated

with 5 year

survival of 73%

AFP usually

normal

HCG elevated

in 10%

seminomas

Lactate

dehydrogenase;

elevated in 10-

20% seminomas

(but also in many

other conditions)

Sheet like lobular

patterns of cells

with substantial

fibrous

component.

Fibrous septa

contain

lymphocytic

inclusions and

granulomas may

be seen.

Non seminomatous germ

cell tumours (42%)

Teratoma

Yolk sac tumour

Choriocarcinoma

Mixed germ cell

tumours (10%)

Younger age at

presentation =20-30

years

Advanced disease

carries worse prognosis

(48% at 5 years)

Retroperitoneal lymph

node dissection may be

needed for residual

disease after

chemotherapy

AFP elevated

in up to 70% of

cases

HCG elevated

in up to 40% of

cases

Other markers

rarely helpful

Heterogenous

texture with

occasional ectopic

tissue such as hair

Image demonstrating a classical seminoma, these tumours are typically more uniform than

teratomas

Page 103: E. F. G. H. I. - 1 File Download

Image sourced from Wikipedia

Risk factors for testicular cancer

Cryptorchidism Infertility

Family history Klinefelter's syndrome Mumps orchitis

Features

Page 104: E. F. G. H. I. - 1 File Download

A painless lump is the most common presenting symptom Pain may also be present in a minority of men Other possible features include hydrocele, gynaecomastia

Diagnosis

Ultrasound is first-line CT scanning of the chest/ abdomen and pelvis is used for staging Tumour markers (see above) should be measured

Management

Orchidectomy (Inguinal approach) Chemotherapy and radiotherapy may be given depending on staging Abdominal lesions >1cm following chemotherapy may require retroperitoneal lymph node

dissection.

Prognosis is generally excellent

5 year survival for seminomas is around 95% if Stage I

5 year survival for teratomas is around 85% if Stage I

Benign disease Epididymo-orchitis Acute epididymitis is an acute inflammation of the epididymis, often involving the testis and usually caused by bacterial infection.

Infection spreads from the urethra or bladder. In men <35 years, gonorrhoea or chlamydia are the usual infections.

Amiodarone is a recognised non infective cause of epididymitis, which resolves on stopping the drug.

Tenderness is usually confined to the epididymis, which may facilitate differentiating it from torsion where pain usually affects the entire testis.

Testicular torsion

Twist of the spermatic cord resulting in testicular ischaemia and necrosis.

Most common in males aged between 10 and 30 (peak incidence 13-15 years) Pain is usually severe and of sudden onset. Cremasteric reflex is lost and elevation of the testis does not ease the pain.

Page 105: E. F. G. H. I. - 1 File Download

Treatment is with surgical exploration. If a torted testis is identified then both testis should be fixed as the condition of bell clapper testis is often bilateral.

Hydrocele

Presents as a mass that transilluminates, usually possible to "get above" it on examination. In younger men it should be investigated with USS to exclude tumour. In children it may occur as a result of a patent processus vaginalis.

Treatment in adults is with a Lords or Jabouley procedure. Treatment in children is with trans inguinal ligation of PPV.

Next question

Page 106: E. F. G. H. I. - 1 File Download

Question 46 of 147

What is the most common cause of osteolytic bone metastasis in children?

Osteosarcoma

Neuroblastoma

Leukaemia

Rhabdomyosarcoma

Nephroblastoma

Neuroblastomas are a relatively common childhood tumour and have a strong tendency to developing widespread lytic metastasis. Please rate this question:

Discuss and give feedback

Next question

Secondary malignant tumours of bone

Metastatic lesions affecting bone are more common than primary bone tumours. The typical tumours that spread to bone include:

Breast

Bronchus Renal Thyroid Prostate

75% cases will affect those over the age of 50 The commonest bone sites affected are:

Page 107: E. F. G. H. I. - 1 File Download

Vertebrae (usually thoracic) Proximal femur Ribs Sternum

Pelvis Skull

Pathological fracture Osteolytic lesions are the greatest risk for pathological fracture The risk and load required to produce fracture varies according to bone site. Bones with lesions that occupy 50% or less will be prone to fracture under loading (Harrington). When 75% of the bone is affected the process of torsion about a bony fulcrum may produce a fracture. The Mirel scoring[1] system may be used to help determine the risk of fracture and is more systematic than the Harrington system described above. Mirel Scoring system

Score

points Site

Radiographic

appearance

Width of bone

involved Pain

1 Upper extremity Blastic Less than 1/3 Mild

2 Lower extremity

Mixed 1/3 to 2/3 Moderate

3 Peritrochanteric Lytic More than 2/3 Aggravated by

function

Depending upon the score the treatment should be as follows:

Score Risk of fracture Treatment

9 or greater Impending (33%) Prophylactic fixation

8 Borderline Consider fixation

7 or less Not impending (4%) Non operative management

Page 108: E. F. G. H. I. - 1 File Download

Where the lesion is an isolated metastatic deposit consideration should be given to excision and reconstruction as the outcome is better [2]. Non operative treatments

Hypercalcaemia- Treat with re hydration and bisphosphonates. Pain- Opiate analgesics and radiotherapy. Some tumours such as breast and prostate will benefit from chemotherapy and or hormonal agents. References 1. Mirels, H. Metastatic disease in long bones. A proposed scoring system for diagnosing impending pathologic fractures. Clin Orthop Relat Res, 1989(249): p. 256-64. 2. Mavrogenis, A.F. et al. Survival analysis of patients with femoral metastases. J Surg Oncol, 2011.

Next question

Page 109: E. F. G. H. I. - 1 File Download

Question 47 of 147

Which of the following features are not typical of Crohns disease?

Complex fistula in ano

Small bowel strictures

Skip lesions

'Rose thorn ulcers' on barium studies

Pseudopolyps on colonoscopy

Pseudopolyps are a feature of ulcerative colitis and occur when there is severe mucosal ulceration. The remaining islands of mucosa may then appear to be isolated and almost polypoidal. Please rate this question:

Discuss and give feedback

Next question

Crohns disease

Crohns disease is a chronic transmural inflammation of a segment(s) of the gastrointestinal tract and may be associated with extra intestinal manifestations. Frequent disease patterns observed include ileal, ileocolic and colonic disease. Peri-anal disease may occur in association with any of these. The disease is often discontinuous in its distribution. Inflammation may cause ulceration, fissures, fistulas and fibrosis with stricturing. Histology reveals a chronic inflammatory infiltrate that is usually patchy and transmural. Ulcerative colitis Vs Crohns

Crohn's disease Ulcerative colitis

Distribution Mouth to anus Rectum and colon

Page 110: E. F. G. H. I. - 1 File Download

Crohn's disease Ulcerative colitis

Macroscopic

changes

Cobblestone appearance, apthoid ulceration Contact bleeding

Depth of disease Transmural inflammation Superficial inflammation

Distribution pattern

Patchy Continuous

Histological

features

Granulomas (non caseating epithelioid cell

aggregates with Langerhans' giant cells)

Crypt abscesses, Inflammatory

cells in the lamina propria

Extraintestinal manifestations of Crohns

Related to disease extent Unrelated to disease extent

Aphthous ulcers (10%) Sacroiliiitis (10-15%)

Erythema nodosum (5-10%) Ankylosing spondylitis (1-2%)

Pyoderma gangrenosum (0.5%) Primary sclerosing cholangitis (Rare)

Acute arthropathy (6-12%) Gallstones (up to 30%)

Ocular complications (up to 10%) Renal calculi (up to 10%)

Diarrhoea in Crohns Diarrhoea in Crohns may be multifactorial since actual inflammation of the colon is not common. Causes therefore include the following:

Bile salt diarrhoea secondary to terminal ileal disease Entero-colic fistula

Short bowel due to multiple resections Bacterial overgrowth

Page 111: E. F. G. H. I. - 1 File Download

Surgical interventions in Crohns disease

The commonest disease pattern in Crohns is stricturing terminal ileal disease and this often culminates in an ileocaecal resection. Other procedures performed include segmental small bowel resections and stricturoplasty. Colonic involvement in patients with Crohns is not common and, where found, distribution is often segmental. However, despite this distribution segmental resections of the colon in patients with Crohns disease are generally not advocated because the recurrence rate in the remaining colon is extremely high, as a result the standard options of colonic surgery in Crohns patients are generally; sub total colectomy, panproctocolectomy and staged sub total colectomy and proctectomy. Restorative procedures such as ileoanal pouch have no role in therapy. Crohns disease is notorious for the developmental of intestinal fistulae; these may form between the rectum and skin (peri anal) or the small bowel and skin. Fistulation between loops of bowel may also occur and result in bacterial overgrowth and malabsorption. Management of enterocutaneous fistulae involves controlling sepsis, optimising nutrition, imaging the disease and planning definitive surgical management.

Next question

Page 112: E. F. G. H. I. - 1 File Download

Question 48 of 147

At which of the following sites is the development of diverticulosis least likely?

Caecum

Ascending colon

Transverse colon

Sigmoid colon

Rectum

Rectal involvement with diverticular disease almost never occurs.

Because the rectum has a circular muscle coat (blending of of the tenia marks the recto-sigmoid junction), diverticular disease almost never occurs here. Right sided colonic diverticular disease is well recognised (though less common than left sided). Please rate this question:

Discuss and give feedback

Next question

Diverticular disease

Diverticular disease is a common surgical problem. It consists of herniation of colonic mucosa through the muscular wall of the colon. The usual site is between the taenia coli where vessels pierce the muscle to supply the mucosa. For this reason, the rectum, which lacks taenia, is often spared. Symptoms

Altered bowel habit Bleeding Abdominal pain

Page 113: E. F. G. H. I. - 1 File Download

Complications

Diverticulitis Haemorrhage

Development of fistula Perforation and faecal peritonitis Perforation and development of abscess Development of diverticular phlegmon

Diagnosis Patients presenting in clinic will typically undergo either a colonoscopy, CT cologram or barium enema as part of their diagnostic work up. All tests can identify diverticular disease. It can be far more difficult to confidently exclude cancer, particularly in diverticular strictures. Acutely unwell surgical patients should be investigated in a systematic way. Plain abdominal films and an erect chest x-ray will identify perforation. An abdominal CT scan (not a CT cologram) with oral and intravenous contrast will help to identify whether acute inflammation is present but also the presence of local complications such as abscess formation.

Severity Classification- Hinchey

I Para-colonic abscess

II Pelvic abscess

III Purulent peritonitis

IV Faecal peritonitis

Treatment

Increase dietary fibre intake.

Mild attacks of diverticulitis may be managed conservatively with antibiotics. Peri colonic abscesses should be drained either surgically or radiologically. Recurrent episodes of acute diverticulitis requiring hospitalisation are a relative indication for

a segmental resection.

Hinchey IV perforations (generalised faecal peritonitis) will require a resection and usually a stoma. This group have a very high risk of post operative complications and usually require HDU admission. Less severe perforations may be managed by laparoscopic washout and drain insertion.

Page 114: E. F. G. H. I. - 1 File Download

Question 49 of 147

A 32 year old lady presents with a 1.5cm pigmented lesion on her back. The surgeon is concerned that this may be a melanoma. What is the most appropriate course of action?

2mm punch biopsy from the centre of the lesion

4mm punch biopsy from the centre of the lesion

Wide excision of the lesion with 3cm margins

Excisional biopsy of the lesion

Wide excision of the lesion with 1cm margins

Suspicious naevi should NOT be partially sampled as histological interpretation is severely compromised. Complete excision is mandatory where lesions fulfil diagnostic criteria. However, wide excision for margins may be deferred until definitive histology is available.

Lesions that are suspicious for melanoma should be excised with complete margins. Radical excision is not routinely undertaken for diagnostic purposes and therefore if subsequent histopathological assessment determines that the lesion is a melanoma a re-exicision of margins may be required. Incisional punch biopsies of potential melanomas makes histological interpretation difficult and is best avoided. Please rate this question:

Discuss and give feedback

Next question

Malignant melanoma

The main diagnostic features (major criteria):

Change in size

Change in shape

Change in colour

Secondary features (minor criteria)

Diameter >6mm

Inflammation

Oozing or bleeding

Page 115: E. F. G. H. I. - 1 File Download

Altered sensation

Treatment

Suspicious lesions should undergo excision biopsy. The lesion should be removed in completely as incision biopsy can make subsequent histopathological assessment difficult.

Once the diagnosis is confirmed the pathology report should be reviewed to determine whether further re-excision of margins is required (see below):

Margins of excision-Related to Breslow thickness

Lesions 0-1mm thick 1cm

Lesions 1-2mm thick 1- 2cm (Depending upon site and pathological features)

Lesions 2-4mm thick 2-3 cm (Depending upon site and pathological features)

Lesions >4 mm thick 3cm

Marsden J et al Revised UK guidelines for management of Melanoma. Br J Dermatol 2010 163:238-256. Further treatments such as sentinel lymph node mapping, isolated limb perfusion and block dissection of regional lymph node groups should be selectively applied.

Next question

Page 116: E. F. G. H. I. - 1 File Download

Question 50 of 147

A 50 year old male presents with painless frank haematuria. Clinical examination is unremarkable. Routine blood tests reveal a haemoglobin of 18g/dl but are otherwise normal. What is the most likely underlying diagnosis?

Squamous cell carcinoma of the bladder

Adenocarcinoma of the prostate

Adenocarcinoma of the kidney

Wilms tumour

Transitional cell carcinoma of the renal pelvis

Theme from April 2012 Exam Polycythaemia is a recognised feature of renal cell carcinoma. Wilms tumours most commonly occur in children. Please rate this question:

Discuss and give feedback

Next question

Haematuria

Causes of haematuria

Trauma Injury to renal tract

Renal trauma commonly due to blunt injury (others penetrating

injuries)

Ureter trauma rare: iatrogenic

Bladder trauma: due to RTA or pelvic fractures

Infection Remember TB

Page 117: E. F. G. H. I. - 1 File Download

Malignancy Renal cell carcinoma (remember paraneoplastic syndromes):

painful or painless

Urothelial malignancies: 90% are transitional cell carcinoma, can

occur anywhere along the urinary tract. Painless haematuria.

Squamous cell carcinoma and adenocarcinoma: rare bladder

tumours

Prostate cancer

Penile cancers: SCC

Renal disease Glomerulonephritis

Stones Microscopic haematuria common

Structural

abnormalities

Benign prostatic hyperplasia (BPH) causes haematuria due to

hypervascularity of the prostate gland

Cystic renal lesions e.g. polycystic kidney disease

Vascular malformations

Renal vein thrombosis due to renal cell carcinoma

Coagulopathy Causes bleeding of underlying lesions

Drugs Cause tubular necrosis or interstitial nephritis: aminoglycosides,

chemotherapy

Interstitial nephritis: penicillin, sulphonamides, and NSAIDs

Anticoagulants

Benign Exercise

Gynaecological Endometriosis: flank pain, dysuria, and haematuria that is cyclical

Iatrogenic Catheterisation

Radiotherapy; cystitis, severe haemorrhage, bladder necrosis

Pseudohaematuria For example following consumption of beetroot

References Http://bestpractice.bmj.com/best-practice/monograph/316/overview/aetiology.html

Next question

Page 118: E. F. G. H. I. - 1 File Download